You are on page 1of 83

Gleim CMA Test Prep: Part 2: Financial Decision Making

(283 questions)

[1] The relevance of a particular cost to a decision is determined by

A. Riskiness of the decision.


B. Number of decision variables.
C. Amount of the cost.
D. Potential effect on the decision.

[2] Of the following decisions, capital budgeting techniques would least likely be used in evaluating the

A. Acquisition of new aircraft by a cargo company.


B. Design and implementation of a major advertising program.
C. Trade for a star quarterback by a football team.
D. Adoption of a new method of allocating nontraceable costs to product lines.

[3] In equipment-replacement decisions, which one of the following does not affect the decision-making process?

A. Current disposal price of the old equipment.


B. Operating costs of the old equipment.
C. Original fair market value of the old equipment.
D. Cost of the new equipment.

[4] The term that refers to costs incurred in the past that are not relevant to a future decision is

A. Discretionary cost.
B. Full absorption cost.
C. Underallocated indirect cost.
D. Sunk cost.

[5] Which one of the following statements concerning cash flow determination for capital budgeting purposes is not correct?

A. Tax depreciation must be considered because it affects cash payments for taxes.
B. Book depreciation is relevant because it affects net income.
C. Sunk costs are not incremental flows and should not be included.
D. Net working capital changes should be included in cash flow forecasts.

[6] A depreciation tax shield is

A. An after-tax cash outflow.


B. A reduction in income taxes.
C. The cash provided by recording depreciation.
D. The expense caused by depreciation.

Copyright 2008 Gleim Publications, Inc. Page 1


Printed for Bahaa Hassan
Gleim CMA Test Prep: Part 2: Financial Decision Making
(283 questions)

[7] Lawson, Inc., is expanding its manufacturing plant, which requires an investment of $4 million in new equipment and plant
modifications. Lawson’s sales are expected to increase by $3 million per year as a result of the expansion. Cash investment in
current assets averages 30% of sales; accounts payable and other current liabilities are 10% of sales. What is the estimated
total investment for this expansion?

A. $3.4 million.
B. $4.3 million.
C. $4.6 million.
D. $5.2 million.

[8] Kline Corporation is expanding its plant, which requires an investment of $8 million in new equipment. Kline’s sales are
expected to increase by $6 million per year as a result of the expansion. Cash investment in current assets averages 30% of
sales, and accounts payable and other current liabilities are 10% of sales. What is the estimated total cash investment for this
expansion?

A. $6.8 million.
B. $8.6 million.
C. $9.2 million.
D. $9.8 million.

[9] Regal Industries is replacing a grinder purchased 5 years ago for $15,000 with a new one costing $25,000 cash. The original
grinder is being depreciated on a straight-line basis over 15 years to a zero salvage value; Regal will sell this old equipment to
a third party for $6,000 cash. The new equipment will be depreciated on a straight-line basis over 10 years to a zero salvage
value. Assuming a 40% marginal tax rate, Regal’s net cash investment at the time of purchase if the old grinder is sold and the
new one purchased is

A. $19,000
B. $15,000
C. $17,400
D. $25,000

[10] Garfield, Inc., is considering a 10-year capital investment project with forecasted revenues of $40,000 per year and forecasted
cash operating expenses of $29,000 per year. The initial cost of the equipment for the project is $23,000, and Garfield expects
to sell the equipment for $9,000 at the end of the tenth year. The equipment will be depreciated over 7 years. The project
requires a working capital investment of $7,000 at its inception and another $5,000 at the end of Year 5. Assuming a 40%
marginal tax rate, the expected net cash flow from the project in the tenth year is

A. $32,000
B. $24,000
C. $20,000
D. $11,000

Copyright 2008 Gleim Publications, Inc. Page 2


Printed for Bahaa Hassan
Gleim CMA Test Prep: Part 2: Financial Decision Making
(283 questions)

[Fact Pattern #1]


The Moore Corporation is considering the acquisition of a new machine. The machine can be purchased for $90,000; it will cost
$6,000 to transport to Moore’s plant and $9,000 to install. It is estimated that the machine will last 10 years, and it is expected to have
an estimated salvage value of $5,000. Over its 10-year life, the machine is expected to produce 2,000 units per year, each with a
selling price of $500 and combined material and labor costs of $450 per unit. Federal tax regulations permit machines of this type to
be depreciated using the straight-line method over 5 years with no estimated salvage value. Moore has a marginal tax rate of 40%.

[11] (Refers to Fact Pattern #1)


What is the net cash outflow at the beginning of the first year that Moore Corporation should use in a capital budgeting
analysis?

A. $(85,000)
B. $(90,000)
C. $(96,000)
D. $(105,000)

[12] (Refers to Fact Pattern #1)


What is the net cash flow for the third year that Moore Corporation should use in a capital budgeting analysis?

A. $68,400
B. $68,000
C. $64,200
D. $79,000

[13] (Refers to Fact Pattern #1)


What is the net cash flow for the tenth year of the project that Moore Corporation should use in a capital budgeting analysis?

A. $100,000
B. $81,000
C. $68,400
D. $63,000

[14] Metrejean Industries is analyzing a capital investment proposal for new equipment to produce a product over the next 8 years.
At the end of 8 years, the equipment must be removed from the plant and will have a net carrying amount of $0, a tax basis of
$150,000, a cost to remove of $80,000, and scrap salvage value of $20,000. Metrejean’s effective tax rate is 40%. What is the
appropriate “end-of-life” cash flow related to these items that should be used in the analysis?

A. $90,000
B. $54,000
C. $24,000
D. $(36,000)

Copyright 2008 Gleim Publications, Inc. Page 3


Printed for Bahaa Hassan
Gleim CMA Test Prep: Part 2: Financial Decision Making
(283 questions)

[15] Kore Industries is analyzing a capital investment proposal for new equipment to produce a product over the next 8 years. The
analyst is attempting to determine the appropriate “end-of-life” cash flows for the analysis. At the end of 8 years, the
equipment must be removed from the plant and will have a net book value of zero, a tax basis of $75,000, a cost to remove of
$40,000, and scrap salvage value of $10,000. Kore’s effective tax rate is 40%. What is the appropriate “end-of-life” cash flow
related to these items that should be used in the analysis?

A. $45,000
B. $27,000
C. $12,000
D. $(18,000)

[Fact Pattern #2]


The Dickins Corporation is considering the acquisition of a new machine at a cost of $180,000. Transporting the machine to Dickins’
plant will cost $12,000. Installing the machine will cost an additional $18,000. It has a 10-year life and is expected to have a salvage
value of $10,000. Furthermore, the machine is expected to produce 4,000 units per year with a selling price of $500 and combined
direct materials and direct labor costs of $450 per unit. Federal tax regulations permit machines of this type to be depreciated using
the straight-line method over 5 years with no estimated salvage value. Dickins has a marginal tax rate of 40%.

[16] (Refers to Fact Pattern #2)


What is the net cash outflow at the beginning of the first year that Dickins should use in a capital budgeting analysis?

A. $(170,000)
B. $(180,000)
C. $(192,000)
D. $(210,000)

[17] (Refers to Fact Pattern #2)


What is the net cash flow for the third year that Dickins should use in a capital budgeting analysis?

A. $136,800
B. $136,000
C. $128,400
D. $107,400

[18] (Refers to Fact Pattern #2)


What is the net cash flow for the tenth year of the project that Dickins should use in a capital budgeting analysis?

A. $200,000
B. $158,000
C. $136,800
D. $126,000

Copyright 2008 Gleim Publications, Inc. Page 4


Printed for Bahaa Hassan
Gleim CMA Test Prep: Part 2: Financial Decision Making
(283 questions)

[Fact Pattern #3]


On January 1, Crane Company will acquire a new asset that costs $400,000 and is anticipated to have a salvage value of $30,000 at
the end of 4 years. The new asset

Qualifies as 3-year property under the Modified Accelerated Cost Recovery System (MACRS).
Will replace an old asset that currently has a tax basis of $80,000 and can be sold now for $60,000.
Will continue to generate the same operating revenues as the old asset ($200,000 per year). However, savings in operating
costs will be experienced as follows: a total of $120,000 in each of the first 3 years and $90,000 in the fourth year.

Crane is subject to a 40% tax rate and rounds all computations to the nearest dollar. Assume that any gain or loss affects the taxes
paid at the end of the year in which it occurred. The company uses the net present value method to analyze projects using the
following factors and rates:

Present Value Present Value of


Period of $1 at 14% $1 Annuity at 14% MACRS
1 .88 .88 33%
2 .77 1.65 45
3 .68 2.33 15
4 .59 2.92 7

[19] (Refers to Fact Pattern #3)


The present value of the depreciation tax shield for the fourth year MACRS depreciation of Crane Company’s new asset is

A. $0
B. $6,112
C. $6,608
D. $16,520

[20] (Refers to Fact Pattern #3)


The discounted net-of-tax amount that should be factored into Crane Company’s analysis for the disposal transaction is

A. $45,760
B. $60,000
C. $67,040
D. $68,000

[21] The accounting rate of return

A. Is synonymous with the internal rate of return.


B. Focuses on income as opposed to cash flows.
C. Is inconsistent with the divisional performance measure known as return on investment.
D. Recognizes the time value of money.

Copyright 2008 Gleim Publications, Inc. Page 5


Printed for Bahaa Hassan
Gleim CMA Test Prep: Part 2: Financial Decision Making
(283 questions)

[22] What is a challenge that the long-term aspect of capital budgeting presents to the management accountant.

A. Activity can be tracked for a single accounting period.


B. Capital projects affect multiple accounting periods.
C. The flexibility of the capital budgeting decision.
D. Freedom of the organization’s financial planning.

[23] Which of the following is not a category of relevant cash flows?

A. Annual net cash flows.


B. Project termination cash flows.
C. Incremental cash flows.
D. Net initial investment.

[24] The capital budgeting process contains several stages. At which stage are financial and nonfinancial factors addressed?

A. Identification and definition.


B. Selection.
C. Search.
D. Information-acquisition.

[25] Book rate of return is an unsatisfactory guide to selecting capital projects because

I. It uses accrual accounting numbers.


II. It compares a single project against the average of capital projects.
III. It uses cash flows to gauge the desirability of the project.

A. I only.
B. I & II.
C. III only.
D. I, II, & III.

[26] The maximum benefit forgone by using a scarce resource for a given purpose and not for the next-best alternative is called

A. Opportunity cost.
B. Sunk cost.
C. Incremental cash flow.
D. Net initial investment.

Copyright 2008 Gleim Publications, Inc. Page 6


Printed for Bahaa Hassan
Gleim CMA Test Prep: Part 2: Financial Decision Making
(283 questions)

[27] Post-investment audits

A. Complete a stage in the capital budgeting process.


B. Serve as a control mechanism.
C. Allow the outcome of a project to be evaluated as soon as possible.
D. Deter managers from proposing profitable investments.

[Fact Pattern #4]


Calamity Cauliflower Corporation is considering undertaking a capital project.
The company would have to commit $24,000 of working capital in addition to an immediate outlay of $160,000 for new equipment.
The project is expected to generate $100,000 of annual income for 10 years. At the end of that time, the new equipment, which will
be depreciated on a straight-line basis, is expected to have a salvage value of $10,000.
The existing equipment that would be sold to make room for the project has a historical cost of $220,000 and accumulated
depreciation of $208,000. It has an estimated remaining useful life of 2 years and the remaining carrying amount is being depreciated
on a straight-line basis. A scrap dealer has agreed to buy it for $8,000.
The company’s effective tax rate is 40%.

[28] (Refers to Fact Pattern #4)


Calamity Cauliflower’s tax benefit arising from the disposal of the old equipment is

A. $8,000
B. $4,800
C. $3,200
D. $1,600

[29] (Refers to Fact Pattern #4)


The total after-tax cash inflow relevant to Calamity Cauliflower’s disposal of the old equipment is

A. $9,600
B. $8,000
C. $6,400
D. $1,600

[30] (Refers to Fact Pattern #4)


The net initial investment required for Calamity Cauliflower to undertake this capital project is

A. $184,000
B. $176,000
C. $174,400
D. $160,000

Copyright 2008 Gleim Publications, Inc. Page 7


Printed for Bahaa Hassan
Gleim CMA Test Prep: Part 2: Financial Decision Making
(283 questions)

[31] (Refers to Fact Pattern #4)


Calamity Cauliflower’s relevant after-tax annual cash inflow from the ongoing operations of the project i.

A. $100,000
B. $60,000
C. $40,000
D. $0

[32] (Refers to Fact Pattern #4)


Calamity Cauliflower’s expected additional depreciation tax shield for the first year of the project i.

A. $10,000
B. $6,400
C. $4,000
D. $2,400

[33] (Refers to Fact Pattern #4)


Calamity Cauliflower’s expected depreciation tax shield for the final year of the project i.

A. $6,400
B. $4,000
C. $2,400
D. $0

[34] (Refers to Fact Pattern #4)


If the project is accepted, Calamity Cauliflower’s expected net cash inflow at the end of the first year i.

A. $110,000
B. $64,000
C. $60,000
D. $56,000

[35] (Refers to Fact Pattern #4)


Calamity Cauliflower’s expected total cash inflow in the final year of the project is

A. $100,400
B. $96,400
C. $90,000
D. $72,400

Copyright 2008 Gleim Publications, Inc. Page 8


Printed for Bahaa Hassan
Gleim CMA Test Prep: Part 2: Financial Decision Making
(283 questions)

[36] A firm is considering a capital project for which the following information is available: An existing piece of equipment that
would be disposed of to make room for new equipment has a historical cost of $360,000. It has a salvage value of $10,000 and
has been depreciated on a straight-line basis for 16 of the estimated 18 years of its useful life. The new equipment has a cost of
$500,000 and the firm expects it will have to devote $20,000 in cash and $24,000 in accounts receivable to the new project.
The firm’s effective tax rate is 40%. The required net initial investment in the new project is

A. $544,000
B. $534,000
C. $522,000
D. $498,000

[37] A firm is considering a new capital project. A salvage company is offering the firm $800,000 for its old equipment. If the firm
accepts the salvage company’s offer, the net initial investment in the project will be

A. Less if the carrying amount is less than the $800,000 offer.


B. Greater if the carrying amount is less than the $800,000 offer.
C. Unaffected by whether a gain or loss is recognized on the disposal.
D. Not determinable from the information given.

[38] Assume that the old equipment, which has not yet been fully depreciated, must be sold in order to purchase the new
equipment. The entity’s policy is to depreciate all equipment on a straight-line basis. Given a constant effective income tax
rate, the incremental depreciation tax shield during the later years of a capital project is generally

A. Greater than that during the earlier years.


B. Less than that during the earlier years.
C. The same as that during the earlier years.
D. Not determinable from the information given.

[39] Which of the following is irrelevant in projecting the cash flows of the final year of a capital project?

A. Cash devoted to use in project.


B. Disposal value of equipment purchased specifically for project.
C. Depreciation tax shield generated by equipment purchased specifically for project.
D. Historical cost of equipment disposed of in the project’s first year.

[40] Capital budgeting is concerned with

A. Decisions affecting only capital intensive industries.


B. Analysis of short-range decisions.
C. Analysis of long-range decisions.
D. Scheduling office personnel in office buildings.

Copyright 2008 Gleim Publications, Inc. Page 9


Printed for Bahaa Hassan
Gleim CMA Test Prep: Part 2: Financial Decision Making
(283 questions)

[41] Capital budgeting is used for the decision analysis of

A. Adding product lines or facilities.


B. Multiple profitable alternatives.
C. Lease-or-buy decisions.
D. All of the answers are correct.

[Fact Pattern #5]


The tax impact of equipment depreciation affects capital budgeting decisions. Currently, the Modified Accelerated Cost Recovery
System (MACRS) is used as the depreciation method for most assets for tax purposes.

[42] (Refers to Fact Pattern #5)


The MACRS method of depreciation for assets with 3, 5, 7, and 10-year recovery periods is most similar to which one of the
following depreciation methods used for financial reporting purposes?

A. Straight-line.
B. Units-of-production.
C. Sum-of-the-years’-digits.
D. 200% declining-balance.

[43] (Refers to Fact Pattern #5)


When employing the MACRS method of depreciation in a capital budgeting decision, the use of MACRS as compared with
the straight-line method of depreciation will result in

A. Equal total depreciation for both methods.


B. MACRS producing less total depreciation than straight line.
C. Equal total tax payments, after discounting for the time value of money.
D. MACRS producing more total depreciation than straight line.

Copyright 2008 Gleim Publications, Inc. Page 10


Printed for Bahaa Hassan
Gleim CMA Test Prep: Part 2: Financial Decision Making
(283 questions)

[44] The U.S. Postal Service is looking for a new machine to help sort the mail. Two companies have submitted bids to Cliff
Kraven, the postal inspector responsible for choosing a machine. A cash flow analysis of the two machines indicates the
following:

Year Machine A Machine B


0 $(30,000) $(30,000)
1 0 13,000
2 0 13,000
3 0 13,000
4 60,000 13,000

If the cost of capital for the Postal Service is 8%, which of the two mail sorters should Cliff choose and why?

A. Machine A, because NPV of A > NPV of B, by $1,044.


B. Machine B, because NPV of A > NPV of B, by $22,000.
C. Machine A, because NPV of A > NPV of B, by $8,000.
D. Machine B, because IRR of A > IRR of B.

[45] The Hopkins Company has estimated that a proposed project’s 10-year annual net cash benefit, received each year end, will be
$2,500 with an additional terminal benefit of $5,000 at the end of the 10th year. Assuming that these cash inflows satisfy
exactly Hopkins’ required rate of return of 8%, calculate the initial cash outlay.

A. $16,775
B. $19,090
C. $25,000
D. $30,000

[46] The chief financial officer of Pauley, Inc., has requested an evaluation of a proposed acquisition of a new machine at a
purchase price of $60,000 and with installation costs of $10,000. A $3,000 increase in working capital will be required. The
machine will have a useful life of 4 years, after which it can be sold for $10,000. The estimated annual incremental operating
revenues and cash operating expenses are $150,000 and $100,000, respectively, for each of the 4 years. Pauley’s effective
income tax rate is 40%, and the cost of capital is 12%. Pauley uses straight-line depreciation for both financial reporting and
income tax purposes.

Pauley’s estimated after-tax cash flow in the fourth year, at which time the equipment will be sold, will b.

A. $37,000
B. $46,000
C. $49,000
D. $50,000

Copyright 2008 Gleim Publications, Inc. Page 11


Printed for Bahaa Hassan
Gleim CMA Test Prep: Part 2: Financial Decision Making
(283 questions)

[47] The management of Pelican, Inc., is evaluating a proposed acquisition of a new machine at a purchase price of $180,000 and
with installation costs of $10,000. A $9,000 increase in working capital will be required. The machine will have a useful life
of 4 years, after which it can be sold for $30,000. The estimated annual incremental operating revenues and cash operating
expenses are $450,000 and $300,000, respectively, for each of the 4 years. Pelican’s effective income tax rate is 40%, and the
cost of capital is 12%. Pelican uses straight-line depreciation for both financial reporting and income tax purposes.

If the project is accepted, the estimated incremental after-tax operating cash flows at the end of the first year will b.

A. $90,000
B. $106,000
C. $109,000
D. $150,000

[48] Mobile Home Manufacturing, Inc., is evaluating a proposed acquisition of a new machine at a purchase price of $380,000 and
installation charges that will amount to $20,000. A $15,000 increase in working capital will be required. The machine will
have a useful life of 4 years, after which it can be sold for $50,000. The estimated annual incremental operating revenues and
cash operating expenses are $750,000 and $500,000, respectively, for each of the 4 years. Mobile Home’s tax rate is 40%, and
the cost of capital is 12%. Mobile Home uses straight-line depreciation for both financial reporting and income tax purposes.
If Mobile Home accepts the project, the initial investment will b.

A. $345,000
B. $365,000
C. $385,000
D. $415,000

[49] A company is considering the purchase of a new machine to replace a five-year old machine and has gathered the following
information:

Purchase price of new machine $50,000


Installation cost of new machine 4,000
Market value (selling price) of the old machine 5,000
Book value of the old machine 2,000
Increase in net working capital if new machine is installed 1,000
Effective income tax rate 40%

If the company replaces the old machine with the new machine, what is the cash flow in period 0.

A. $(49,000)
B. $(51,200)
C. $(51,800)
D. $(53,000)

Copyright 2008 Gleim Publications, Inc. Page 12


Printed for Bahaa Hassan
Gleim CMA Test Prep: Part 2: Financial Decision Making
(283 questions)

[50] Which one of the following capital investment evaluation methods does not take the time value of money into consideration?

A. Net present value.


B. Discounted payback.
C. Internal rate of return.
D. Accounting rate of return.

[51] Capital investment projects include proposals for all of the following except

A. The acquisition of government mandated pollution control equipment.


B. The expansion of existing product offerings.
C. Additional research and development facilities.
D. Refinancing existing working capital agreements.

[52] Which one of the following items is least likely to directly impact an equipment replacement capital expenditure decision?

A. The net present value of the equipment that is being replaced.


B. The depreciation rate that will be used for tax purposes on the new asset.
C. The amount of additional accounts receivable that will be generated from increased production and sales.
D. The sales value of the asset that is being replaced.

[53] Cora Lewis is performing an analysis to determine if her firm should invest in new equipment to produce a product recently
developed by her firm. The option would be to abandon the product. She uses the net present value (NPV) method and
discounts at the firm’s cost of capital. Cora is contemplating how to handle the following items:

I. The book value of warehouse space currently used by another division.


II. Interest payments on debt to finance the equipment.
III. Increased levels of accounts payable and inventory.
IV. R&D spent in prior years and treated as a deferred asset for book and tax purposes.

Which of the above items are relevant for Cora to consider in determining the cash flows for her NPV calculation?

A. I, II, III and IV.


B. II and III only.
C. IV only.
D. III and IV only.

Copyright 2008 Gleim Publications, Inc. Page 13


Printed for Bahaa Hassan
Gleim CMA Test Prep: Part 2: Financial Decision Making
(283 questions)

[Fact Pattern #6]


Calvin, Inc., is considering the purchase of a new state-of-the-art machine to replace its hand-operated machine. Calvin’s effective tax
rate is 40%, and its cost of capital is 12%. Data regarding the existing and new machines are presented below.

Existing New
Machine Machine
Original cost $50,000 $90,000
Installation costs 0 4,000
Freight and insurance 0 6,000
Expected end salvage value 0 0
Depreciation method straight-line straight-line
Expected useful life 10 years 5 years

[54] (Refers to Fact Pattern #6)


The existing machine has been in service for 7 years and could be sold currently for $25,000. Calvin expects to realize a
before-tax annual reduction in labor costs of $30,000 if the new machine is purchased and placed in service.
If the new machine is purchased, the incremental cash flows for the fifth year would amount to

A. $18,000
B. $24,000
C. $26,000
D. $30,000

[55] (Refers to Fact Pattern #6)


The existing machine has been in service for 7 years and could be sold currently for $25,000. If the new machine is purchased,
Calvin expects to realize a $30,000 before-tax annual reduction in labor costs.
If the new machine is purchased, what is the net amount of the initial cash outflow at time = 0 for net present value calculation
purposes?

A. $65,000
B. $75,000
C. $79,000
D. $100,000

[56] (Refers to Fact Pattern #6)


The existing machine has been in service for 5 years and could be sold currently for $25,000. Calvin expects to realize annual
before-tax reductions in labor costs of $30,000 if the new machine is purchased and placed in service.
If the new machine is purchased, the incremental cash flows for the first year would amount to

A. $18,000
B. $24,000
C. $30,000
D. $45,000

Copyright 2008 Gleim Publications, Inc. Page 14


Printed for Bahaa Hassan
Gleim CMA Test Prep: Part 2: Financial Decision Making
(283 questions)

[Fact Pattern #7]


Olson Industries needs to add a small plant to accommodate a special contract to supply building materials over a 5-year period. The
required initial cash outlays at time = 0 are as follows:

Land $ 500,000
New building 2,000,000
Equipment 3,000,000

Olson uses straight-line depreciation for tax purposes and will depreciate the building over 10 years and the equipment over 5 years.
Olson’s effective tax rate is 40%. Revenues from the special contract are estimated at $1.2 million annually, and cash expenses are
estimated at $300,000 annually. At the end of the fifth year, the assumed sales values of the land and building are $800,000 and
$500,000, respectively. It is further assumed the equipment will be removed at a cost of $50,000 and sold for $300,000.

[57] (Refers to Fact Pattern #7)


As Olson utilizes the net present value (NPV) method to analyze investments, the net cash flow for Period 3 would be

A. $60,000
B. $860,000
C. $880,000
D. $940,000

[58] (Refers to Fact Pattern #7)


As Olson utilizes the net present value (NPV) method to analyze investments, the net cash flow for Period 5 would be

A. $1,710,000
B. $2,070,000
C. $2,230,000
D. $2,390,000

Copyright 2008 Gleim Publications, Inc. Page 15


Printed for Bahaa Hassan
Gleim CMA Test Prep: Part 2: Financial Decision Making
(283 questions)

[59] The following schedule reflects the incremental costs and revenues for a capital project. The company uses straight-line
depreciation. The interest expense reflects an allocation of interest on the amount of this investment, which is based on the
company’s weighted average cost of capital.

Revenues $650,000
Direct costs $270,000
Variable overhead 50,000
Fixed overhead 20,000
Depreciation 70,000
General & administrative 40,000
Interest expense 8,000
Total costs (458,000)
Net profits before taxes $192,000

The annual cash flow from this investment, before tax considerations, would be

A. $192,000
B. $200,000
C. $262,000
D. $270,000

[Fact Pattern #8]


Kell, Inc., is analyzing an investment for a new product expected to have annual sales of 100,000 units for the next 5 years and then
be discontinued. New equipment will be purchased for $1,200,000 and cost $300,000 to install. The equipment will be depreciated on
a straight-line basis over 5 years for financial reporting purposes and 3 years for tax purposes. At the end of the fifth year, it will cost
$100,000 to remove the equipment, which can be sold for $300,000. Additional working capital of $400,000 will be required
immediately and needed for the life of the product. The product will sell for $80, with direct labor and material costs of $65 per unit.
Annual indirect costs will increase by $500,000. Kell’s effective tax rate is 40%.

[60] (Refers to Fact Pattern #8)


In a capital budgeting analysis, what is the expected cash flow at time = 5 (fifth year of operations) that Kell should use to
compute the net present value?

A. $720,000
B. $800,000
C. $1,120,000
D. $1,240,000

Copyright 2008 Gleim Publications, Inc. Page 16


Printed for Bahaa Hassan
Gleim CMA Test Prep: Part 2: Financial Decision Making
(283 questions)

[61] (Refers to Fact Pattern #8)


In a capital budgeting analysis, what is the cash flow at time = 0 (initial investment) that Kell should use to compute the net
present value?

A. $1,300,000
B. $1,500,000
C. $1,700,000
D. $1,900,000

[62] (Refers to Fact Pattern #8)


In a capital budgeting analysis, what is the expected cash flow at time = 3 (third year of operation) that Kell should use to
compute the net present value?

A. $300,000
B. $720,000
C. $760,000
D. $800,000

[Fact Pattern #9]


Skytop Industries is analyzing a capital investment project using discounted cash flow (DCF) analysis. The new equipment will cost
$250,000. Installation and transportation costs aggregating $25,000 will be capitalized. Annual incremental pre-tax cash inflows are
estimated at $75,000. Skytop’s effective income tax rate is 40%.

[63] (Refers to Fact Pattern #9)


In this scenario, Skytop will employ a 5-year MACRS depreciation schedule (20%, 32%, 19.2%, 11.52%, 11.52%, 5.76%)
with the half-year convention. Existing equipment, with a book value of $100,000 and an estimated market value of $80,000,
will be sold immediately after installation of the new equipment. After-tax cash flow for the first year of the project would
amount to

A. $45,000
B. $52,000
C. $67,000
D. $75,000

[64] (Refers to Fact Pattern #9)


In this scenario, Skytop will immediately sell existing equipment after installation of the new equipment. The existing
equipment has a tax basis of $100,000 and an estimated market value of $80,000. Skytop estimates that the new equipment’s
capacity will generate additional receivables and inventory of $30,000, while payables will increase by $15,000. Total after-
tax cash outflows occurring in Year 0 would b.

A. $177,000
B. $182,000
C. $198,000
D. $202,000

Copyright 2008 Gleim Publications, Inc. Page 17


Printed for Bahaa Hassan
Gleim CMA Test Prep: Part 2: Financial Decision Making
(283 questions)

[65] (Refers to Fact Pattern #9)


Skytop’s appropriate 5-year depreciation schedule (20%, 32%, 19%, 14.5%, 14.5%) will be employed with no terminal value
factored into the computations. Assuming the machine is sold at the end of Year 5 for $30,000, the after-tax cash flow for
Year 5 of the project would amount to

A. $63,950
B. $72,950
C. $78,950
D. $86,925

[66] Regis Company, which is subject to an effective income tax rate of 30%, is evaluating a proposed capital project. Relevant
information for the proposed project is summarized below.

Initial investment $500,000


Annual operating cash inflows
for the first 3 years:
Year 1 185,000
Year 2 175,000
Year 3 152,000

Depreciation will be calculated under the straight-line method using an 8-year estimated service life and a terminal value of
$50,000. In determining the estimated total after-tax cash flow in Year 2 of the project, Regis should consider the after-tax
operating cash

A. Inflow only.
B. Inflow plus annual depreciation expense.
C. Inflow plus annual depreciation tax shield.
D. Inflow plus the net impact of the annual depreciation expense and depreciation tax shield.

Copyright 2008 Gleim Publications, Inc. Page 18


Printed for Bahaa Hassan
Gleim CMA Test Prep: Part 2: Financial Decision Making
(283 questions)

[67] Fuller Industries is considering a $1 million investment in stamping equipment to produce a new product. The equipment is
expected to last 9 years, produce revenue of $700,000 per year, and have related cash expenses of $450,000 per year. At the
end of the ninth year, the equipment is expected to have a salvage value of $100,000 and cost $50,000 to remove. The IRS
categorizes this as 5-year Modified Accelerated Cost Recovery System (MACRS) property subject to the following
depreciation rates.

Year Rate
1 20.00%
2 32.00%
3 19.20%
4 11.52%
5 11.52%
6 5.76%

Fuller’s effective income tax rate is 40%, and Fuller expects, on an overall company basis, to continue to be profitable and
have significant taxable income. If Fuller uses the net present value method to analyze investments, what is the expected net
tax impact on cash flow in Year 2 before discounting.

A. Positive $28,000 impact.


B. $0 impact.
C. Negative $100,000 impact.
D. Negative $128,000 impact.

[68] In estimating “after-tax incremental cash flows” under discounted cash flow analysis for capital project evaluations, which one
of the following options reflects the items that should be included in the analyses?

Project related Estimated


Sunk changes in impacts
Costs net working capital on inflation
A. No No Yes
B. No Yes Yes
C. No Yes No
D. Yes No No

Copyright 2008 Gleim Publications, Inc. Page 19


Printed for Bahaa Hassan
Gleim CMA Test Prep: Part 2: Financial Decision Making
(283 questions)

[69] AGC Company is considering an equipment upgrade. AGC uses discounted cash flow (DCF) analysis in evaluating capital
investments and has an effective tax rate of 40%. Selected data developed by AGC is as follows:

Existing New
Equipment Equipment
Original cost $50,000 $95,000
Accumulated depreciation 45,000 --
Current market value 3,000 95,000
Accounts receivable 6,000 8,000
Accounts payable 2,100 2,500

Based on this information, what is the initial investment for a DCF analysis of this proposed upgrade.

A. $92,400
B. $92,800
C. $95,800
D. $96,200

[70] In discounted cash flow techniques, which one of the following alternatives best reflects the items to be incorporated in the
initial net cash investment.

Net proceeds
Capitalized Changes from sale of Impact of
expenditures in net old asset in spontaneous
(e.g., shipping working a replacement changes in
costs) capital decision current liabilities
A. No Yes Yes Yes
B. Yes No No No
C. No Yes No No
D. Yes Yes Yes Yes

[71] The owner of Woofie’s Video Rental cannot decide how to project the real costs of opening a rental store in a new shopping
mall. The owner knows the capital investment required but is not sure of the returns from a store in a new mall. Historically,
the video rental industry has had an inflation rate equal to the economic norm. The owner requires a real internal rate of return
of 10%. Inflation is expected to be 3% during the next few years. The industry expects a new store to show a growth rate,
without inflation, of 8%. First year revenues at the new store are expected to be $400,000.
The revenues for the second year, using both the real rate approach and the nominal rate approach, respectively, would b.

A. $432,000 real and $444,960 nominal.


B. $432,000 real and $452,000 nominal.
C. $440,000 real and $452,000 nominal.
D. $440,000 real and $453,200 nominal.

Copyright 2008 Gleim Publications, Inc. Page 20


Printed for Bahaa Hassan
Gleim CMA Test Prep: Part 2: Financial Decision Making
(283 questions)

[72] The stage of the capital budgeting process that has the most risk is

A. Identifying alternative possible projects.


B. Forecasting cash flow.
C. Raising funds to initially support the project.
D. Evaluating performance and learning.

[73] Charles Company owns a building that originally cost $400,000 and has a current book value of $250,000. The building was
financed by a loan that has one payment of $20,000 outstanding, which must be paid off upon the sale of the building. Charles
Company would like to purchase a new building for $600,000. If the new building is purchased, the existing building would
be sold for $380,000. Charles Company’s income tax rate is 40%. If the new building is purchased, the relevant initial cash
flows would total

A. $272,000
B. $292,000
C. $372,000
D. $392,000

[74] A project with a 4-year life has a cost of acquisition of $400,000 and installation cost of $100,000. If the effective income tax
rate is 40%, what is the cash inflow each period due to depreciation expense?

A. $40,000
B. $50,000
C. $60,000
D. $75,000

[75] The net present value (NPV) method of investment project analysis assumes that the project’s cash flows are reinvested at the

A. Computed internal rate of return.


B. Risk-free interest rate.
C. Discount rate used in the NPV calculation.
D. Firm’s accounting rate of return.

[76] The rankings of mutually exclusive investments determined using the internal rate of return method (IRR) and the net present
value method (NPV) may be different when

A. The lives of the multiple projects are equal and the size of the required investments are equal.
B. The required rate of return equals the IRR of each project.
C. The required rate of return is higher than the IRR of each project.
D. Multiple projects have unequal lives and the size of the investment for each project is different.

Copyright 2008 Gleim Publications, Inc. Page 21


Printed for Bahaa Hassan
Gleim CMA Test Prep: Part 2: Financial Decision Making
(283 questions)

[Fact Pattern #10]


A firm with an 18% desired rate of return is considering the following projects (on January 1, Year 1):

January 1, Year 1 December 31, Year 5


Cash Outflow Cash Inflow Project Internal
(000’s Omitted) (000’s Omitted) Rate of Return
Project A $3,500 $7,400 16%
Project B 4,000 9,950 ?

Present Value of $1 Due at the End of N Periods


N 12% 14% 15% 16% 18% 20% 22%
4 .6355 .5921 .5718 .5523 .5158 .4823 .4230
5 .5674 .5194 .4972 .4761 .4371 .4019 .3411
6 .5066 .4556 .4323 .4104 .3704 .3349 .2751

[77] (Refers to Fact Pattern #10)


Using the net-present-value (NPV) method, Project A’s net present value is

A. $316,920
B. $23,140
C. $(265,460)
D. $(316,920)

[78] (Refers to Fact Pattern #10)


Project B’s internal rate of return is closest to

A. 15%
B. 16%
C. 18%
D. 20%

[79] Amster Corporation has not yet decided on its hurdle rate for use in the evaluation of capital budgeting projects. This lack of
information will prohibit Amster from calculating a project’s

Accounting Net Internal


Rate of Present Rate of
Return Value Return
A. No No No
B. Yes Yes Yes
C. No Yes Yes
D. No Yes No

Copyright 2008 Gleim Publications, Inc. Page 22


Printed for Bahaa Hassan
Gleim CMA Test Prep: Part 2: Financial Decision Making
(283 questions)

[80] All of the following items are included in discounted cash flow analysis except

A. Future operating cash savings.


B. The current asset disposal price.
C. The future asset depreciation expense.
D. The tax effects of future asset depreciation.

[81] The use of an accelerated method instead of the straight-line method of depreciation in computing the net present value of a
project has the effect of

A. Raising the hurdle rate necessary to justify the project.


B. Lowering the net present value of the project.
C. Increasing the present value of the depreciation tax shield.
D. Increasing the cash outflows at the initial point of the project.

[82] The NPV of a project has been calculated to be $215,000. Which one of the following changes in assumptions would decrease
the NPV?

A. Decrease the estimated effective income tax rate.


B. Decrease the initial investment amount.
C. Extend the project life and associated cash inflows.
D. Increase the discount rate.

[83] A disadvantage of the net present value method of capital expenditure evaluation is that it

A. Is calculated using sensitivity analysis.


B. Computes the true interest rate.
C. Does not provide the true rate of return on investment.
D. Is difficult to apply because it uses a trial-and-error approach.

Copyright 2008 Gleim Publications, Inc. Page 23


Printed for Bahaa Hassan
Gleim CMA Test Prep: Part 2: Financial Decision Making
(283 questions)

[Fact Pattern #11]


In order to increase production capacity, Gunning Industries is considering replacing an existing production machine with a new
technologically improved machine effective January 1. The following information is being considered by Gunning Industries:

• The new machine would be purchased for $160,000 in cash. Shipping, installation, and testing would cost an additional
$30,000.
• The new machine is expected to increase annual sales by 20,000 units at a sales price of $40 per unit. Incremental operating
costs include $30 per unit in variable costs and total fixed costs of $40,000 per year.
• The investment in the new machine will require an immediate increase in working capital of $35,000. This cash outflow will
be recovered after 5 years.
• Gunning uses straight-line depreciation for financial reporting and tax reporting purposes. The new machine has an
estimated useful life of 5 years and zero salvage value.
• Gunning is subject to a 40% corporate income tax rate.

Gunning uses the net present value method to analyze investments and will employ the following factors and rates:

Present Value Present Value of an Ordinary


Period of $1 at 10% Annuity of $1 at 10%
1 .909 .909
2 .826 1.736
3 .751 2.487
4 .683 3.170
5 .621 3.791

[84] (Refers to Fact Pattern #11)


Gunning Industries’ net cash outflow in a capital budgeting decision is

A. $190,000
B. $195,000
C. $204,525
D. $225,000

[85] (Refers to Fact Pattern #11)


Gunning Industries’ discounted annual depreciation tax shield for the year of replacement is

A. $13,817
B. $16,762
C. $20,725
D. $22,800

Copyright 2008 Gleim Publications, Inc. Page 24


Printed for Bahaa Hassan
Gleim CMA Test Prep: Part 2: Financial Decision Making
(283 questions)

[86] (Refers to Fact Pattern #11)


The acquisition of the new production machine by Gunning Industries will contribute a discounted net-of-tax contribution
margin of

A. $242,624
B. $303,280
C. $363,936
D. $454,920

[87] (Refers to Fact Pattern #11)


The overall discounted cash flow impact of Gunning Industries’ working capital investment for the new production machine
would be

A. $(7,959)
B. $(10,080)
C. $(13,265)
D. $(35,000)

[88] Jackson Corporation uses net present value techniques in evaluating its capital investment projects. The company is
considering a new equipment acquisition that will cost $100,000, fully installed, and have a zero salvage value at the end of its
five-year productive life. Jackson will depreciate the equipment on a straight-line basis for both financial and tax purposes.
Jackson estimates $70,000 in annual recurring operating cash income and $20,000 in annual recurring operating cash
expenses. Jackson’s desired rate of return is 12% and its effective income tax rate is 40%. What is the net present value of this
investment on an after-tax basis?

A. $28,840
B. $8,150
C. $36,990
D. $80,250

[89] A weakness of the internal rate of return (IRR) approach for determining the acceptability of investments is that it

A. Does not consider the time value of money.


B. Is not a straightforward decision criterion.
C. Implicitly assumes that the firm is able to reinvest project cash flows at the firm’s cost of capital.
D. Implicitly assumes that the firm is able to reinvest project cash flows at the project’s internal rate of return.

[90] The internal rate of return (IRR) is the

A. Hurdle rate.
B. Rate of interest for which the net present value is greater than 1.0.
C. Rate of interest for which the net present value is equal to zero.
D. Rate of return generated from the operational cash flows.

Copyright 2008 Gleim Publications, Inc. Page 25


Printed for Bahaa Hassan
Gleim CMA Test Prep: Part 2: Financial Decision Making
(283 questions)

[91] Suzie owns a computer reselling business and is expanding it. She is presented with two options. Under Proposal A, the
estimated investment for the expansion project is $85,000, and it is expected to produce after-tax cash flows of $25,000 for
each of the next 6 years. Proposal B involves an investment of $32,000 and after-tax cash flows of $10,000 for each of the
next 6 years. Between which two desired rates of return will Suzie be indifferent to either proposal?

A. 10% and 12%.


B. 14% and 16%.
C. 16% and 18%.
D. 18% and 20%.

[92] The net present value method of capital budgeting assumes that cash flows are reinvested at

A. The risk-free rate.


B. The cost of debt.
C. The rate of return of the project.
D. The discount rate used in the analysis.

[93] The net present value of a proposed investment is negative; therefore, the discount rate used must be

A. Greater than the project’s internal rate of return.


B. Less than the project’s internal rate of return.
C. Greater than the firm’s cost of equity.
D. Less than the risk-free rate.

[94] Dr. G invested $10,000 in a lifetime annuity for his granddaughter Emily. The annuity is expected to yield $400 annually
forever. What is the anticipated internal rate of return for the annuity?

A. Cannot be determined without additional information.


B. 4.0%
C. 2.5%
D. 8.0%

[95] Which of the following statements is most likely correct for a project costing $50,000 and returning $14,000 per year for 5
years?

A. NPV = $36,274.
B. NPV = $20,000.
C. IRR = 1.4%.
D. IRR is greater than 10%.

Copyright 2008 Gleim Publications, Inc. Page 26


Printed for Bahaa Hassan
Gleim CMA Test Prep: Part 2: Financial Decision Making
(283 questions)

[96] What is the approximate IRR for a project that costs $50,000 and provides cash inflows of $20,000 for 3 years?

A. 10%
B. 12%
C. 22%
D. 27%

[97] Pena Company is considering a project that calls for an initial cash outlay of $50,000. The expected net cash inflows from the
project are $7,791 for each of 10 years. What is the IRR of the project?

A. 6%
B. 7%
C. 8%
D. 9%

[98] The following forecasts have been prepared for a new investment by Oxford Industries of $20 million with an 8-year life:

Pessimistic Expected Optimistic


Market size 60,000 90,000 140,000
Market share, % 25 30 35
Unit price $750 $800 $875
Unit variable cost $500 $400 $350
Fixed cost, millions $7 $4 $3.5

Assume that Oxford employs straight-line depreciation, and that they are taxed at 35%. Assuming an opportunity cost of
capital of 14%, what is the NPV of this project, based on expected outcomes?

A. $2,626,415
B. $4,563,505
C. $6,722,109
D. $8,055,722

[99] Drillers, Inc., is evaluating a project to produce a high-tech deep-sea oil exploration device. The investment required is $80
million for a plant with a capacity of 15,000 units a year for 5 years. The device will be sold for a price of $12,000 per unit.
Sales are expected to be 12,000 units per year. The variable cost is $7,000 and fixed costs, excluding depreciation, are $25
million per year. Assume Drillers employs straight-line depreciation on all depreciable assets, and assume that they are taxed
at a rate of 36%. If the required rate of return is 12%, what is the approximate NPV of the project?

A. $17,225,000
B. $21,511,000
C. $26,780,000
D. $56,124,800

Copyright 2008 Gleim Publications, Inc. Page 27


Printed for Bahaa Hassan
Gleim CMA Test Prep: Part 2: Financial Decision Making
(283 questions)

[100] Assume that the probability distribution of NPVs is normal. The firm considers true risk occurring if the project results in a
NPV that is zero or less. If the expected NPV is $1,000 and the standard deviation of NPV is $500, what is the probability
that the project has an NPV of 0 or less?

A. Less than 3%.


B. Greater than 3%, but less than 9%.
C. Greater than 9%, but less than 16%.
D. Greater than 16%.

[101] Project 1 has an expected NPV of $120,000 and a standard deviation of $200,000. Project 2 has an expected NPV of
$100,000 and a standard deviation of $150,000. The correlation between these two projects is 0.80. What is the coefficient of
variation for the portfolio of projects?

A. 1.67
B. 1.59
C. 1.51
D. 0.63

[102] When using the net present value method for capital budgeting analysis, the required rate of return is called all of the
following except the

A. Risk-free rate.
B. Cost of capital.
C. Discount rate.
D. Cutoff rate.

[103] The internal rate of return for a project can be determined

A. If the internal rate of return is greater than the firm’s cost of capital.
B. Only if the project cash flows are constant.
C. By finding the discount rate that yields a net present value of zero for the project.
D. By subtracting the firm’s cost of capital from the project’s profitability index.

[104] Carco, Inc., wants to use discounted cash flow techniques when analyzing its capital investment projects. The company is
aware of the uncertainty involved in estimating future cash flows. A simple method some companies employ to adjust for the
uncertainty inherent in their estimates is to

A. Prepare a direct analysis of the probability of outcomes.


B. Use accelerated depreciation.
C. Adjust the minimum desired rate of return.
D. Increase the estimates of the cash flows.

Copyright 2008 Gleim Publications, Inc. Page 28


Printed for Bahaa Hassan
Gleim CMA Test Prep: Part 2: Financial Decision Making
(283 questions)

[105] The accountant of Ronier, Inc., has prepared an analysis of a proposed capital project using discounted cash flow techniques.
One manager has questioned the accuracy of the results because the discount factors employed in the analysis have assumed
the cash flows occurred at the end of the year when the cash flows actually occurred uniformly throughout each year. The net
present value calculated by the accountant will

A. Not be in error.
B. Be slightly overstated.
C. Be unusable for actual decision making.
D. Be slightly understated but usable.

[106] The internal rate of return on an investment

A. Usually coincides with the company’s hurdle rate.


B. Disregards discounted cash flows.
C. May produce different rankings from the net present value method on mutually exclusive projects.
D. Would tend to be reduced if a company used an accelerated method of depreciation for tax purposes rather than the
straight-line method.

[107] The internal rate of return is

A. The breakeven borrowing rate for the project in question.


B. The yield rate/effective rate of interest quoted on long-term debt and other instruments.
C. Favorable when it exceeds the hurdle rate.
D. All of the answers are correct.

[108] All of the following are the rates used in net present value analysis except for the

A. Cost of capital.
B. Hurdle rate.
C. Discount rate.
D. Accounting rate of return.

[109] In evaluating a capital budget project, the use of the net present value (NPV) model is generally not affected by the

A. Method of funding the project.


B. Initial cost of the project.
C. Amount of added working capital needed for operations during the term of the project.
D. Project’s salvage value.

Copyright 2008 Gleim Publications, Inc. Page 29


Printed for Bahaa Hassan
Gleim CMA Test Prep: Part 2: Financial Decision Making
(283 questions)

[110] An advantage of the net present value method over the internal rate of return model in discounted cash flow analysis is that the
net present value method

A. Computes a desired rate of return for capital projects.


B. Can be used when there is no constant rate of return required for each year of the project.
C. Uses a discount rate that equates the discounted cash inflows with the outflows.
D. Uses discounted cash flows whereas the internal rate of return model does not.

[111] Basic time value of money concepts concern

Interest Factors Risk Cost of capital


A. Yes Yes No
B. Yes No Yes
C. No Yes No
D. No No Yes

[112] The present value may be calculated for discounted cash

Inflows Outflows Annuities


A. Yes Yes Yes
B. Yes No Yes
C. No Yes No
D. No No Yes

[113] Assume that the interest rate is greater than zero. Which of the following cash-inflow streams should you prefer?

Year 1 Year 2 Year 3 Year 4


A. $400 $300 $200 $100
B. $100 $200 $300 $400
C. $250 $250 $250 $250
D. Any of these, since they each sum to $1,000.

Copyright 2008 Gleim Publications, Inc. Page 30


Printed for Bahaa Hassan
Gleim CMA Test Prep: Part 2: Financial Decision Making
(283 questions)

[114] Wilkinson, Inc., which has a cost of capital of 12%, invested in a project with an internal rate of return (IRR) of 14%. The
project is expected to have a useful life of four years, and it will produce net cash inflows as follows:

Year Net Cash Inflows


1 $1,000
2 2,000
3 4,000
4 4,000

The initial cost of this project amounted t.

A. $7,483
B. $8,530
C. $11,000
D. $12,540

[115] Gibber Corporation has an opportunity to sell a newly developed product in the United States for a period of 5 years. The
product license would be purchased from NewGroup Company. Gibber would be responsible for all distribution and product
promotion costs. NewGroup has the option to renew the agreement, with modifications, at the end of the initial 5-year term.
Gibber has developed the following estimated revenues and costs that would be associated with the new product:

Cost of new equipment required $120,000


Additional working capital required 200,000
Salvage value of equipment in Year 5 20,000
Annual revenues and costs.
Sales revenues 400,000
Costs of goods sold 250,000
Out-of-pocket operating cost 70,000

The working capital required to support the new product would be released for investment elsewhere if the product licensing
agreement is not renewed.

Using the net present value method of analysis and ignoring the effects of income taxes, the net present value of this product
agreement, assuming Gibber has a 20% cost of capital, would b.

A. $7,720
B. $(64,064)
C. $(72,680)
D. $(127,320)

Copyright 2008 Gleim Publications, Inc. Page 31


Printed for Bahaa Hassan
Gleim CMA Test Prep: Part 2: Financial Decision Making
(283 questions)

[116] Bennet, Inc., uses the net present value method to evaluate capital projects. Bennet’s required rate of return is 10%. Bennet is
considering two mutually exclusive projects for its manufacturing business. Both projects require an initial outlay of $120,000
and are expected to have a useful life of four years. The projected after-tax cash flows associated with these projects are as
follows:

Year Project X Project Y


1 $ 40,000 $ 10,000
2 40,000 20,000
3 40,000 60,000
4 40,000 80,000
Total $160,000 $170,000

Assuming adequate funds are available, which of the following project options would you recommend that Bennet’s
management undertake.

A. Project X only.
B. Project Y only.
C. Projects X and Y.
D. Neither project.

[117] Brown and Company uses the internal rate of return (IRR) method to evaluate capital projects. Brown is considering four
independent projects with the following IRRs:

Project IRR
I 10%
II 12%
III 14%
IV 15%

Brown’s cost of capital is 13%. Which one of the following project options should Brown accept based on IRR?

A. Projects I and II only.


B. Projects III and IV only.
C. Project IV only.
D. Projects I, II, III and IV.

Copyright 2008 Gleim Publications, Inc. Page 32


Printed for Bahaa Hassan
Gleim CMA Test Prep: Part 2: Financial Decision Making
(283 questions)

[Fact Pattern #12]

The following data pertain to a 4-year project being Metro is subject to a 40% income tax rate and rounds all
considered by Metro Industries: computations to the nearest dollar. Assume that any gain or loss
affects the taxes paid at the end of the year in which it occurred.
• A depreciable asset that costs $1,200,000 will be The company uses the net present value method to analyze
acquired on January 1. The asset, which is expected to investments and will employ the following factors and rates.
have a $200,000 salvage value at the end of 4 years,
qualifies as 3-year property under the Modified Present Value Present Value of
Accelerated Cost Recovery System (MACRS). Period of $1 at 12% $1 Annuity at 12% MACRS
• The new asset will replace an existing asset that has a 1 0.89 0.89 33%
tax basis of $150,000 and can be sold on the same 2 0.80 1.69 45
January 1 for $180,000. 3 0.71 2.40 15
4 0.64 3.04 7
• The project is expected to provide added annual sales
of 30,000 units at $20. Additional cash operating costs
are: variable, $12 per unit; fixed, $90,000 per year.
• A $50,000 working capital investment that is fully
recoverable at the end of the fourth year is required.

[118] (Refers to Fact Pattern #12)


The discounted cash flow for the fourth year MACRS depreciation on the new asset is

A. $0
B. $17,920
C. $21,504
D. $26,880

[119] (Refers to Fact Pattern #12)


The discounted, net-of-tax amount that relates to disposal of the existing asset is

A. $168,000
B. $169,320
C. $180,000
D. $190,680

[120] (Refers to Fact Pattern #12)


The expected incremental sales will provide a discounted, net-of-tax contribution margin over 4 years of

A. $57,600
B. $92,160
C. $273,600
D. $437,760

Copyright 2008 Gleim Publications, Inc. Page 33


Printed for Bahaa Hassan
Gleim CMA Test Prep: Part 2: Financial Decision Making
(283 questions)

[121] (Refers to Fact Pattern #12)


The overall discounted-cash-flow impact of the working capital investment on Metro’s project is

A. $(2,800)
B. $(18,000)
C. $(50,000)
D. $(59,200)

[122] A company is considering two mutually exclusive projects with the following projected cash flows:

Year Project A Project B


0 $(100,000) $(100,000)
1 0 34,320
2 0 34,320
3 0 34,320
4 $ 157,352 34,320

The company has a required rate of return of 8%. If the company’s objective is to maximize shareholder wealth, which one of
the following is the most valid reason for selecting one of the projects.

A. The net present value of Project A is greater than the net present value of Project B, therefore select Project A.
B. The net present value of Project A is less than the net present value of Project B, therefore select Project B.
C. The internal rate of return of Project A is greater than the internal rate of return of Project B, therefore select Project A.
D. The internal rate of return of Project A is less than the internal rate of return of Project B, therefore select Project B.

[123] Depreciation is incorporated explicitly in the discounted cash flow analysis of an investment proposal because it

A. Is a cost of operations that cannot be avoided.


B. Is a cash inflow.
C. Reduces the cash outlay for income taxes.
D. Represents the initial cash outflow spread over the life of the investment.

[124] The method that recognizes the time value of money by discounting the after-tax cash flows over the life of a project, using
the company’s minimum desired rate of return is the

A. Accounting rate of return method.


B. Net present value method.
C. Internal rate of return method.
D. Payback method.

Copyright 2008 Gleim Publications, Inc. Page 34


Printed for Bahaa Hassan
Gleim CMA Test Prep: Part 2: Financial Decision Making
(283 questions)

[125] A company has unlimited capital funds to invest. The decision rule for the company to follow in order to maximize
shareholders’ wealth is to invest in all projects having a(n)

A. Present value greater than zero.


B. Net present value greater than zero.
C. Internal rate of return greater than zero.
D. Accounting rate of return greater than the hurdle rate used in capital budgeting analyses.

[126] Future, Inc., is in the enviable situation of having unlimited capital funds. The best decision rule, in an economic sense, for it
to follow would be to invest in all projects in which the

A. Accounting rate of return is greater than the earnings as a percent of sales.


B. Payback reciprocal is greater than the internal rate of return.
C. Internal rate of return is greater than zero.
D. Net present value is greater than zero.

[127] The technique that recognizes the time value of money by discounting the after-tax cash flows for a project over its life to time
period zero using the company’s minimum desired rate of return is called the

A. Net present value method.


B. Payback method.
C. Average rate of return method.
D. Accounting rate of return method.

[128] Mintz Corporation is considering the acquisition of a new technologically efficient packaging machine at a cost of $300,000.
The equipment requires an immediate, fully recoverable investment in working capital of $40,000. Mintz plans to use the
machine for 5 years, is subject to a 40% income tax rate, and uses a 12% hurdle rate when analyzing capital investments. The
company employs the net present value method (NPV) to analyze projects.
The overall impact of the working capital investment on Mintz’s NPV analysis is

A. $(10,392)
B. $(13,040)
C. $(17,320)
D. $(40,000)

[129] For each of the next 6 years, Atlantic Motors anticipates a net income of $10,000, a straight-line tax depreciation of $20,000,
a 40% tax rate, a discount rate of 10%, and cash sales of $100,000. The depreciable assets are all being acquired at the
beginning of Year 1 and will have a salvage value of zero at the end of 6 years. The present value of the total depreciation tax
savings would be

A. $8,000
B. $27,072
C. $34,840
D. $87,100

Copyright 2008 Gleim Publications, Inc. Page 35


Printed for Bahaa Hassan
Gleim CMA Test Prep: Part 2: Financial Decision Making
(283 questions)

[130] Webster Products is performing a capital budgeting analysis on a new product it is considering. Annual sales are expected to
be 50,000 units in the first year, 100,000 units in the second year, and 125,000 units the year thereafter. Selling price will be
$80 in the first year and is expected to decrease by 5% per year. Annual costs are forecasted as follows:

Fixed costs $300,000 each year


Labor cost per unit $20 in year 1, increasing 5% per year thereafter
Material cost per unit $30 in year 1, increasing 10% per year thereafter

The investment of $2 million will be depreciated on a straight-line basis over 4 years for financial reporting and tax purposes.
Webster’s effective tax rate is 40%. When calculating net present value (NPV), the net cash flow for Year 3 would b.

A. $558,750
B. $858,750
C. $1,058,750
D. $1,070,00.

[131] Allstar Company invests in a project with expected cash inflows of $9,000 per year for 4 years. All cash flows occur at year
end. The required return on investment is 9%. If the project generates a net present value (NPV) of $3,000, what is the amount
of the initial investment in the project?

A. $11,25.
B. $13,236
C. $26,160
D. $29,160

[132] Topeka Products uses the net present value (NPV) method to evaluate capital projects. Topeka plans to acquire a depreciable
asset on January 1 of next year for $2.4 million. The new asset has an estimated service life of 4 years, a zero terminal disposal
value, and will be depreciated on a straight-line basis. The new asset will replace an existing asset that is expected to be sold
for $350,000. The tax basis of the existing asset is $330,000. Topeka is subject to an effective income tax rate of 40% and
assumes that any gains or losses affect the taxes paid at the end of the year in which the gains or losses occur. Topeka uses a
10% discount rate for NPV analyses.
The amount related to the new asset’s depreciation that would be included in an NPV analysis is

A. $760,800
B. $1,141,200
C. $1,639,200
D. $1,902,000

Copyright 2008 Gleim Publications, Inc. Page 36


Printed for Bahaa Hassan
Gleim CMA Test Prep: Part 2: Financial Decision Making
(283 questions)

[133] Sarah Birdsong has prepared a net present value (NPV) analysis for a 15-year equipment modernization program. Her initial
calculations include a series of depreciation tax savings, which are then discounted. Birdsong is now considering the
incorporation of inflation into the NPV analysis. If the depreciation tax savings were based on original equipment cost, which
of the following options correctly shows how she should handle the program’s cash operating costs and the firm’s required
rate of return, respectively?

Cash Operating Costs Required Rate of Return


A. Adjust for inflation Adjust for inflation
B. Adjust for inflation Do not adjust for inflation
C. Do not adjust for inflation Adjust for inflation
D. Do not adjust for inflation Do not adjust for inflation

[134] If the present value of expected cash inflows from a project equals the present value of expected cash outflows, the discount
rate is th.

A. Payback rate.
B. Internal rate of return.
C. Accounting rate of return.
D. Net present value rate.

[135] Nolan Hospital has decided to acquire diagnostic equipment from Weber Medical Products based on Weber’s reputation for
quality. Weber has offered Nolan four payment options, as shown in the following table. All payments would be due and paid
at the beginning of each year.

Options
Year (dollars in millions)
I II III IV
1 $5 $18
2 5 $10
3 5 10
4 5 $22

Nolan’s cost of funds is 8%. Which payment option should Nolan choose?

A. I
B. II
C. III
D. IV

Copyright 2008 Gleim Publications, Inc. Page 37


Printed for Bahaa Hassan
Gleim CMA Test Prep: Part 2: Financial Decision Making
(283 questions)

[Fact Pattern #13]


Stennet Company is considering two mutually exclusive projects. The net present value (NPV) profiles of the two projects are as
follows:

Discount Rate Net Present Value $(000)


(percent) Project A Project B
0 $2,220 $1,240
10 681 507
12 495 411
14 335 327
16 197 252
18 77 186
20 (26) 128
22 (115) 76
24 (193) 30
26 (260) (11)
28 (318) (47)

[136] (Refers to Fact Pattern #13)


The approximate internal rates of return for Projects A and B, respectively, are

A. 0% and 0%.
B. 19.0% and 21.5%.
C. 19.5% and 25.5%.
D. 20.5% and 26.5%.

[137] (Refers to Fact Pattern #13)


The company president is of the view that Project B should be accepted because it has the higher internal rate of return (IRR).
The president requested John Mack, the CFO, to make a recommendation. The company’s cost of capital is 10%. Which one
of the following options should Mack recommend to the president.

A. Agree with the president.


B. Accept Project A because it has an IRR higher than that of Project B.
C. Accept both Projects A and B, as the IRR for each project is greater than cost of capital.
D. Accept Project A because at a 10% discount rate, it has an NPV that is greater than that of Project B.

[138] Bell Delivery Co. is financing a new truck with a loan of $30,000 to be repaid in five annual installments of $7,900 at the end
of each year. What is the approximate annual interest rate Bell is paying?

A. 4%
B. 5%
C. 10%
D. 16%

Copyright 2008 Gleim Publications, Inc. Page 38


Printed for Bahaa Hassan
Gleim CMA Test Prep: Part 2: Financial Decision Making
(283 questions)

[139] All of the following are methods used to evaluate investments for capital budgeting decisions except

A. Accounting rate of return.


B. Internal rate of return
C. Excess present value (profitability) index.
D. Required rate of return.

[140] For a given investment project, the interest rate at which the present value of the cash inflows equals the present value of the
cash outflows is called the

A. Hurdle rate.
B. Payback rate.
C. Internal rate of return.
D. Cost of capital.

[141] The net present value of an investment project represents the

A. Total actual cash inflows minus the total actual cash outflows.
B. Excess of the discounted cash inflows over the discounted cash outflows.
C. Total after-tax cash flow including the tax shield from depreciation.
D. Cumulative accounting profit over the life of the project.

[142] Kunkle Products is analyzing whether to invest in equipment to manufacture a new product. The equipment will cost $1
million, is expected to last 10 years, and will be depreciated on a straight-line basis for both financial reporting and tax
purposes. Kunkle’s effective tax rate is 40%, and its hurdle rate is 14%. Other information concerning the project is as
follows:

Sales per year = 10,000 units


Selling price = $100 per unit
Variable cost = $70 per unit

A 10% reduction in variable costs would result in the net present value increasing by approximatel.

A. $156,000
B. $219,000
C. $365,000
D. $367,000

Copyright 2008 Gleim Publications, Inc. Page 39


Printed for Bahaa Hassan
Gleim CMA Test Prep: Part 2: Financial Decision Making
(283 questions)

[143] Wilcox Corporation won a settlement in a law suit and was offered four different payment alternatives by the defendant’s
insurance company. A review of interest rates indicates that 8% is appropriate for analyzing this situation. Ignoring any tax
considerations, which one of the following four alternatives should the controller recommend to Wilcox management.

A. $135,000 now.
B. $40,000 per year at the end of each of the next 4 years.
C. $5,000 now and $20,000 per year at the end of each of the next 10 years.
D. $5,000 now and $5,000 per year at the end of each of the next 9 years, plus a lump-sum payment of $200,000 at the
end of the tenth year.

[Fact Pattern #14]


Smithco is considering the acquisition of scanning equipment to mechanize its procurement process. The equipment will require
extensive testing and debugging, as well as user training, prior to its operational use. Projected after-tax cash flows are shown below.

After-Tax Cash
Year Inflow/(Outflow)
0 $(550,000)
1 (500,000)
2 450,000
3 350,000
4 350,000
5 350,000

Management anticipates the equipment will be sold at the beginning of Year 6 for $50,000 when its book value is zero. Smithco’s
internal hurdle and effective tax rates are 14% and 40%, respectively.

[144] (Refers to Fact Pattern #14)


Smithco’s net present value for the project would be

A. $(1,780)
B. $(6,970)
C. $(17,350)
D. $8,600

[145] (Refers to Fact Pattern #14)


Smithco’s payback period for the project will be

A. 2.3 years.
B. 3.0 years.
C. 3.5 years.
D. 3.75 years.

Copyright 2008 Gleim Publications, Inc. Page 40


Printed for Bahaa Hassan
Gleim CMA Test Prep: Part 2: Financial Decision Making
(283 questions)

[146] Verla Industries is trying to decide which one of the following two options to pursue. Either option will take effect on January
1 of the next year.
Option One - Acquire a New Finishing Machine.

The cost of the machine is $1,000,000, and it will have a useful life of 5 years. Net pre-tax cash flows arising from savings in
labor costs will amount to $100,000 per year for 5 years. Depreciation expense will be calculated using the straight-line
method for both financial and tax reporting purposes. As an incentive to purchase, Verla will receive a trade-in allowance of
$50,000 on their current fully depreciated finishing machine.

Option Two - Outsource the Finishing Work.

Verla can outsource the work to LM, Inc., at a cost of $200,000 per year for 5 years. If they outsource, Verla will scrap their
current fully depreciated finishing machine.

Verla’s effective income tax rate is 40%. The weighted-average cost of capital is 10%.

Verla’s net present value of outsourcing the finishing work i.

A. $303,280 net cash outflow.


B. $404,920 net cash outflow.
C. $454,920 net cash outflow.
D. $758,200 net cash outflow.

[147] Verla Industries is trying to decide which one of the following two options to pursue. Either option will take effect on January
1st of the next year.
Option One - Acquire a New Finishing Machine.

The cost of the machine is $1,000,000, and it will have a useful life of 5 years. Net pre-tax cash flows arising from savings in
labor costs will amount to $100,000 per year for 5 years. Depreciation expense will be calculated using the straight-line
method for both financial and tax reporting purposes. As an incentive to purchase, Verla will receive a trade-in allowance of
$50,000 on their current fully depreciated finishing machine.

Option Two - Outsource the Finishing Work.

Verla can outsource the work to LM, Inc., at a cost of $200,000 per year for 5 years. If they outsource, Verla will scrap their
current fully depreciated finishing machine.

Verla’s effective income tax rate is 40%. The weighted-average cost of capital is 10%.

Verla’s net present value of acquiring the new finishing machine is

A. $229,710 net cash outflow.


B. $267,620 net cash outflow.
C. $369,260 net cash outflow.
D. $434,424 net cash outflow.

Copyright 2008 Gleim Publications, Inc. Page 41


Printed for Bahaa Hassan
Gleim CMA Test Prep: Part 2: Financial Decision Making
(283 questions)

[148] Jenson Copying Company is planning to buy a copying machine costing $25,310. The net present values (NPV) of this
investment, at various discount rates, are as follows:

Discount Rate NPV


4% $2,440
6% 1,420
8% 460
10% (440)

Jenson’s approximate internal rate of return on this investment is

A. 6%
B. 8%
C. 9%
D. 10%

[149] Long, Inc., is analyzing a $1 million investment in new equipment to produce a product with a $5 per-unit margin. The
equipment will last 5 years, be depreciated on a straight-line basis for tax purposes, and have no value at the end of its life. A
study of unit sales produced the following data:

Annual
Unit Sales Probability
80,000 .10
85,000 .20
90,000 .30
95,000 .20
100,000 .10
110,000 .10

If Long utilizes a 12% hurdle rate and is subject to a 40% effective income tax rate, the expected net present value of the
project would b.

A. $261,750
B. $283,380
C. $297,800
D. $427,580

Copyright 2008 Gleim Publications, Inc. Page 42


Printed for Bahaa Hassan
Gleim CMA Test Prep: Part 2: Financial Decision Making
(283 questions)

[150] Fred Kratz just completed a capital investment analysis for the acquisition of new material handling equipment. The
equipment is expected to cost $1,000,000 and be used for 8 years. Kratz reviewed the net present value (NPV) analysis with
Bill Dolan, Vice President of Finance. The analysis shows that the tax shield for this investment has a positive NPV of
$200,000, using the firm’s hurdle rate of 20%. Dolan noticed that 8-year straight-line depreciation was used for tax purposes,
but since this equipment qualifies for 3-year MACRS treatment, the tax shield analysis should be revised. The company has an
effective tax rate of 40%. The MACRS rates for 3-year property are as follows:

Year Rate
1 33.33%
2 44.45%
3 14.81%
4 7.41%

Accordingly, the revised NPV for the tax shield (rounded to the nearest thousand) should b.

A. $109,000
B. $192,000
C. $283,000
D. $425,000

[Fact Pattern #15]


Foster Manufacturing is analyzing a capital investment project that is forecasted to produce the following cash flows and net income:

After-Tax Net
Year Cash Flows Income
0 $(20,000) $ 0
1 6,000 2,000
2 6,000 2,000
3 8,000 2,000
4 8,000 2,000

Foster’s cost of capital is 12%.

[151] (Refers to Fact Pattern #15)


Foster’s net present value for this project is

A. $(1,600.
B. $924
C. $6,07.
D. $6,998

Copyright 2008 Gleim Publications, Inc. Page 43


Printed for Bahaa Hassan
Gleim CMA Test Prep: Part 2: Financial Decision Making
(283 questions)

[152] (Refers to Fact Pattern #15)


Foster’s internal rate of return (rounded to the nearest whole percentage) i.

A. 5.
B. 12%
C. 14%
D. 40.

[153] Lunar, Inc., is considering the purchase of a machine for $500,000 that will last 5 years. A financial analysis is being
developed using the following information:

Year 1 Year 2 Year 3 Year 4 Year 5


Unit sales 10,000 10,000 20,000 20,000 20,000
Selling price per unit $ 100 $ 100 $ 100 $ 100 $ 100
Variable cost per unit 65 65 65 65 65
Fixed costs 300,000 300,000 300,000 300,000 300,000
Pre-tax cash flow 50,000 50,000 400,000 400,000 400,000

The machine will be depreciated over 5 years on a straight-line basis for tax purposes, and Lunar is subject to a 40% effective
income tax rate. Assuming Lunar will have significant taxable income from other lines of business, and using a 20% discount
rate, the net present value of the project will b.

A. $(282,470.
B. $(103,070)
C. $(14,010)
D. $16,530

[154] Foster Manufacturing is analyzing a capital investment project that is forecasted to produce the following cash flows and net
income:

After-Tax Net
Year Cash flow Income
0 ($20,000) $ 0
1 6,000 2,000
2 6,000 2,000
3 8,000 2,000
4 8,000 2,000

Foster’s payback period for this project will b.

A. 2.5 years.
B. 2.6 years.
C. 3.0 years.
D. 3.3 years.

Copyright 2008 Gleim Publications, Inc. Page 44


Printed for Bahaa Hassan
Gleim CMA Test Prep: Part 2: Financial Decision Making
(283 questions)

[155] An investment decision is acceptable if th.

A. Net present value is greater than or equal to $0.


B. Present value of cash inflows is less than the present value of cash outflows.
C. Present value of cash outflows is greater than or equal to $0.
D. Present value of cash inflows is greater than or equal to $0.

[156] Logan Enterprises is at a critical decision point and must decide whether to go out of business or continue to operate for 5
more years. Logan has a labor contract with 5 years remaining that calls for $1.5 million in severance pay if Logan’s plant
shuts down. The firm also has a contract to supply 150,000 units per year, at a price of $100 each, to Dill, Inc., for the next 5
years. Dill is Logan’s only remaining customer. Logan must pay Dill $500,000 immediately if it defaults on the contract. The
plant has a net book value of $600,000, and appraisers estimate the facility would sell for $750,000 today but would have no
market value if operated for another 5 years. Logan’s fixed costs are $4 million per year, and variable costs are $75 per unit.
Logan’s appropriate discount rate is 12%. Ignoring taxes, the optimal decision is t.

A. Shut down because the annual cash flow is negative $250,000 per year.
B. Keep operating to avoid the severance pay of $1,500,000.
C. Shut down since the breakeven point is 160,000 units, while annual sales are 150,000 units.
D. Keep operating since the incremental net present value is approximately $350,000.

[157] Parker Industries is analyzing a $200,000 equipment investment to produce a new product for the next 5 years. A study of
expected annual after-tax cash flows from the project produced the following data:

Annual
After-Tax
Cash Flow Probability
$45,000 .10
50,000 .20
55,000 .30
60,000 .20
65,000 .10
70,000 .10

If Parker utilizes a 14% hurdle rate, the probability of achieving a positive net present value i.

A. 20%
B. 30%
C. 40%
D. 60%

[158] Which of the following is not a shortcoming of the internal rate of return (IRR) method.

A. IRR assumes that funds generated from a project will be reinvested at an interest rate equal to the project’s IRR.
B. IRR does not take into account the difference in the scale of investment alternatives.
C. IRR is easier to visualize and interpret than net present value (NPV).
D. Sign changes in the cash flow stream can generate more than one IRR.

Copyright 2008 Gleim Publications, Inc. Page 45


Printed for Bahaa Hassan
Gleim CMA Test Prep: Part 2: Financial Decision Making
(283 questions)

[159] A company is in the process of evaluating a major product line expansion. Using a 14% discount rate, the firm has calculated
the present value of both the project’s cash inflows and cash outflows to be $15.8 million. The company will likely evaluate
this project further b.

A. Taking a closer look at the expansion’s contribution margin.


B. Comparing the internal rate of return versus the accounting rate of return.
C. Comparing the internal rate of return versus the company’s cost of capital.
D. Comparing the internal rate of return versus the company’s cost of capital and hurdle rate.

[160] With regard to a capital investment project, which one of the following statements best describes the relationship between the
cost of capital and the expected internal rate of return?

A. The internal rate of return must exceed the cost of capital for the project to be acceptable.
B. If the internal rate of return exceeds zero, the project will be profitable.
C. The cost of capital must exceed the internal rate of return for the project to be acceptable.
D. The internal rate of return should be compared to a pre-determined benchmark without regard to the cost of capital.

[161] National, Inc., is considering three mutually exclusive projects. Each project would involve an initial investment of $7,000
and generate the following cash inflows.

Year Project X Project Y Project Z


1 $ 0 $ 2,000 $ 5,000
2 4,000 3,000 0
3 2,000 4,000 2,000
4 5,000 2,000 4,000
Total $11,000 $11,000 $11,000

Given a cost of capital of 10%, rank the projects in descending order of net present value (NPV).

A. X, Y, Z.
B. X, Z, Y.
C. Y, Z, X.
D. Z, Y, X.

[162] The primary advantage of using the internal rate of return (IRR) method to evaluate capital budgeting projects is that it

A. Results in decisions that will maximize shareholder wealth.


B. Results in decisions that will maximize income.
C. Is easy to understand and communicate.
D. Assumes a conservative reinvestment rate.

Copyright 2008 Gleim Publications, Inc. Page 46


Printed for Bahaa Hassan
Gleim CMA Test Prep: Part 2: Financial Decision Making
(283 questions)

[163] A company invested $500,000 in a new project. The project is expected to yield annual incremental cash flows of $175,000
for 4 years. What is the approximate internal rate of return (IRR) for this project?

A. 10%
B. 15%
C. 35%
D. 40%

[164] One disadvantage of using internal rate of return (IRR) is that it

A. Provides a result that cannot be compared to other projects.


B. May not be used when cash flows vary from positive to negative in different years.
C. Is difficult for managers to understand the results of the calculation.
D. Can only use a limited number of years in calculating the result.

[165] The Financial Analysis Department of Stover, Inc., has analyzed a proposed capital investment and calculated the appropriate
incremental cash flows as follows:

Year Cash Flow


0 $(100,000) outflow
1 80,000 inflow
2 80,000 inflow
3 80,000 inflow
4 (100,000) outflow

A net present value (NPV) of approximately $25,000 and an internal rate of return (IRR) of minus 29% were calculated for
the project, and the project was submitted to the board of directors for approval. Which one of the following statements is
correct?

A. The project has another IRR in addition to the minus 29% rate.
B. The IRR calculation must have contained an error.
C. In the NPV calculation, the project’s cash flows are assumed to be reinvested at Stover’s cost of capital.
D. The board of directors should not approve the project.

[166] Assume that an investment project’s assumed cash flows are not changed, but the assumed weighted-average cost of capital is
reduced. What impact would this have on the net present value (NPV) and the internal rate of return (IRR) of this project?

A. NPV would increase, and IRR would increase.


B. NPV would decrease, and IRR would increase.
C. NPV would not change, and IRR would not change.
D. NPV would increase, and IRR would not change.

Copyright 2008 Gleim Publications, Inc. Page 47


Printed for Bahaa Hassan
Gleim CMA Test Prep: Part 2: Financial Decision Making
(283 questions)

[167] The Eat-Right Company has been disappointed by previous capital budgeting decisions using the payback method. A new
requirement has been implemented that requires discounted cash flow analysis to be used to compute the net present value
(NPV) of proposed purchases over $300,000. The Food Processing Department of the Eat-Right Company is considering the
acquisition of a new machine that will reduce labor costs by a pre-tax amount of $175,000 per year. Other information
regarding the possible acquisition is as follows:

The machine will cost $450,000. Installation charges will amount to an additional $25,000.
The machine will have a useful life of 3 years, with no salvage value. Depreciation rates for tax purposes are 25%,
38%, and 37% for Years 1, 2, and 3, respectively.
Eat-Right’s cost of capital, 12%, is considered the appropriate discount rate.
The income tax rate is 40%.
Cash flows are assumed to occur at the end of the calendar year, which coincides with Eat-Right’s fiscal year end.

Which of the following best indicates the net present value of the proposed investment and the appropriate acquisition
decision?

A. Approximately $73,000; recommend making the investment.


B. Approximately $(73,000); recommend not making the investment.
C. Approximately $55,000; recommend making the investment.
D. Approximately $(55,000); recommend not making the investment.

[Fact Pattern #16]


Henderson, Inc., has purchased a new fleet of trucks to deliver its merchandise. The trucks have a useful life of 8 years and cost a
total of $500,000. Henderson expects its net increase in after-tax cash flow to be $150,000 in Year 1, $175,000 in Year 2, $125,000
in Year 3, and $100,000 in each of the remaining years.

[168] (Refers to Fact Pattern #16)


Ignoring the time value of money, how long will it take Henderson to recover the amount of investment?

A. 3.5 years.
B. 4.0 years.
C. 4.2 years.
D. 5 years.

[169] (Refers to Fact Pattern #16)


What is the payback reciprocal for Henderson’s fleet of trucks?

A. 29%
B. 25%
C. 24%
D. 20%

Copyright 2008 Gleim Publications, Inc. Page 48


Printed for Bahaa Hassan
Gleim CMA Test Prep: Part 2: Financial Decision Making
(283 questions)

[170] (Refers to Fact Pattern #16)


If the net cash flow is $130,000 a year, what is the payback time for Henderson’s fleet of trucks?

A. 3 years.
B. 3.15 years.
C. 3.85 years.
D. 4 years.

[171] (Refers to Fact Pattern #16)


Based on a 6% annual interest rate, what is the discounted payback period for Henderson’s fleet of trucks?

A. 3.5 years.
B. 3.98 years.
C. 4.25 years.
D. 5.0 years.

[172] Jasper Company has a payback goal of 3 years on new equipment acquisitions. A new sorter is being evaluated that costs
$450,000 and has a 5-year life. Straight-line depreciation will be used; no salvage is anticipated. Jasper is subject to a 40%
income tax rate. To meet the company’s payback goal, the sorter must generate reductions in annual cash operating costs of

A. $60,000
B. $100,000
C. $150,000
D. $190,000

[173] The capital budgeting model that is generally considered the best model for long-range decision making is the

A. Payback model.
B. Accounting rate of return model.
C. Unadjusted rate of return model.
D. Discounted cash flow model.

[174] When evaluating projects, breakeven time is best described as

A. Annual fixed costs ÷ monthly contribution margin.


B. Project investment ÷ annual net cash inflows.
C. The point where cumulative cash inflows on a project equal total cash outflows.
D. The point at which discounted cumulative cash inflows on a project equal discounted total cash outflows.

Copyright 2008 Gleim Publications, Inc. Page 49


Printed for Bahaa Hassan
Gleim CMA Test Prep: Part 2: Financial Decision Making
(283 questions)

[175] Irwinn Co. is considering an investment in a capital project. The sole outlay will be $800,000 at the outset of the project, and
the annual net after-tax cash inflow will be $216,309.75 for 6 years. The present value factors at Irwinn’s 8% cost of capital
are

Year PV Factors
1 .926
2 .857
3 .794
4 .735
5 .681
6 .630

What is the breakeven time (BET)?

A. 3.70 years.
B. 4.57 years.
C. 5.00 years.
D. 6.00 years.

[176] A characteristic of the payback method (before taxes) is that it

A. Incorporates the time value of money.


B. Neglects total project profitability.
C. Uses accrual accounting inflows in the numerator of the calculation.
D. Uses the estimated expected life of the asset in the denominator of the calculation.

[177] The length of time required to recover the initial cash outlay of a capital project is determined by using the

A. Discounted cash flow method.


B. Payback method.
C. Weighted net present value method.
D. Net present value method.

[178] Which one of the following statements about the payback method of investment analysis is correct? The payback method

A. Does not consider the time value of money.


B. Considers cash flows after the payback has been reached.
C. Uses discounted cash flow techniques.
D. Generally leads to the same decision as other methods for long-term projects.

Copyright 2008 Gleim Publications, Inc. Page 50


Printed for Bahaa Hassan
Gleim CMA Test Prep: Part 2: Financial Decision Making
(283 questions)

[179] The payback reciprocal can be used to approximate a project’s

A. Profitability index.
B. Net present value.
C. Accounting rate of return if the cash flow pattern is relatively stable.
D. Internal rate of return if the cash flow pattern is relatively stable.

[180] The bailout payback method

A. Incorporates the time value of money.


B. Equals the recovery period from normal operations.
C. Eliminates the disposal value from the payback calculation.
D. Measures the risk if a project is terminated.

[181] Whatney Co. is considering the acquisition of a new, more efficient press. The cost of the press is $360,000, and the press has
an estimated 6-year life with zero salvage value. Whatney uses straight-line depreciation for both financial reporting and
income tax reporting purposes and has a 40% corporate income tax rate. In evaluating equipment acquisitions of this type,
Whatney uses a goal of a 4-year payback period. To meet Whatney’s desired payback period, the press must produce a
minimum annual before-tax operating cash savings of

A. $90,000
B. $110,000
C. $114,000
D. $150,000

[Fact Pattern #17]


The Dickins Corporation is considering the acquisition of a new machine at a cost of $180,000. Transporting the machine to Dickins’
plant will cost $12,000. Installing the machine will cost an additional $18,000. It has a 10-year life and is expected to have a salvage
value of $10,000. Furthermore, the machine is expected to produce 4,000 units per year with a selling price of $500 and combined
direct materials and direct labor costs of $450 per unit. Federal tax regulations permit machines of this type to be depreciated using
the straight-line method over 5 years with no estimated salvage value. Dickins has a marginal tax rate of 40%.

[182] (Refers to Fact Pattern #17)


What is the approximate payback period on Dickins’ new machine?

A. 1.05 years.
B. 1.54 years.
C. 1.33 years.
D. 2.22 years.

Copyright 2008 Gleim Publications, Inc. Page 51


Printed for Bahaa Hassan
Gleim CMA Test Prep: Part 2: Financial Decision Making
(283 questions)

[183] Fitzgerald Company is planning to acquire a $250,000 machine that will provide increased efficiencies, thereby reducing
annual operating costs by $80,000. The machine will be depreciated by the straight-line method over a 5-year life with no
salvage value at the end of 5 years. Assuming a 40% income tax rate, the machine’s payback period is

A. 3.13 years.
B. 8.33 years.
C. 3.68 years.
D. 5.21 years.

[184] When ranking two mutually exclusive investments with different initial amounts, management should give first priority to the
project

A. That generates cash flows for the longer period of time.


B. Whose net after-tax flows equal the initial investment.
C. That has the greater accounting rate of return.
D. That has the greater profitability index.

[185] The bailout payback method

A. Is used by firms with federally insured loans.


B. Calculates the payback period using the sum of the net cash flows and the salvage value.
C. Calculates the payback period using the difference between net cash inflow and the salvage value.
D. Estimates short-term profitability.

Copyright 2008 Gleim Publications, Inc. Page 52


Printed for Bahaa Hassan
Gleim CMA Test Prep: Part 2: Financial Decision Making
(283 questions)

[Fact Pattern #18]


Jorelle Company’s financial staff has been requested to review a proposed investment in new capital equipment. Applicable financial
data is presented below. There will be no salvage value at the end of the investment’s life and, due to realistic depreciation practices,
it is estimated that the salvage value and net book value are equal at the end of each year. All cash flows are assumed to take place at
the end of each year. For investment proposals, Jorelle uses a 12% after-tax target rate of return.

Investment Proposal
Purchase Cost Annual Net Annual
Year and Book Value After-Tax Cash Flows Net Income
0 $250,000 $ 0 $ 0
1 168,000 120,000 35,000
2 100,000 108,000 39,000
3 50,000 96,000 43,000
4 18,000 84,000 47,000
5 0 72,000 51,000

Discounted Factors for a 12% Rate of Return


Present Value of $1.00 Present Value of an Annuity of $1.00
Year Received at the End of Each Period Received at the End of Each Period
1 .89 .89
2 .80 1.69
3 .71 2.40
4 .64 3.04
5 .57 3.61
6 .51 4.12

[186] (Refers to Fact Pattern #18)


The accounting rate of return on the average investment proposal is

A. 12.0%
B. 17.2%
C. 28.0%
D. 34.4%

[187] (Refers to Fact Pattern #18)


The net present value for the investment proposal is

A. $106,160
B. $(97,970)
C. $356,160
D. $96,560

Copyright 2008 Gleim Publications, Inc. Page 53


Printed for Bahaa Hassan
Gleim CMA Test Prep: Part 2: Financial Decision Making
(283 questions)

[188] (Refers to Fact Pattern #18)


The traditional payback period for the investment proposal is

A. Over 5 years.
B. 2.23 years.
C. 1.65 years.
D. 2.83 years.

[189] The following methods are used to evaluate capital investment projects:

Internal rate of return


Average rate of return
Payback
Net present value

Which one of the following correctly identifies the methods that utilize discounted cash-flow (DCF) techniques?

Internal Rate Average Rate Net Present


of Return of Return Payback Value
A. Yes Yes No No
B. No No Yes Yes
C. Yes No Yes No
D. Yes No No Yes

[190] Which one of the following methods for evaluating capital projects is the least useful from an investment analysis point of
view?

A. Accounting rate of return.


B. Internal rate of return.
C. Net present value.
D. Payback.

Copyright 2008 Gleim Publications, Inc. Page 54


Printed for Bahaa Hassan
Gleim CMA Test Prep: Part 2: Financial Decision Making
(283 questions)

[191] Two mutually exclusive capital expenditure projects have the following characteristics:

Project A Project B
Investment $100,000 $150,000
Net cash inflows:
Year 1 40,000 80,000
Year 2 50,000 70,000
Year 3 60,000 60,000

All cash flows are received at the end of the year. Based on this information, which one of the following statements is not
correct?

A. The net present value of Project A at a cost of capital of 10% is $22,720.


B. The net present value of Project B at a cost of capital of 12% is $19,950.
C. The internal rate of return of Project B is greater than the internal rate of return of Project A.
D. The payback period for Project A is longer than the payback period for Project B.

[192] Jones & Company is considering the acquisition of scanning equipment to mechanize its procurement process. The equipment
will require extensive testing and debugging as well as user training prior to its operational use. Projected after-tax cash flows
are as follows:

Time Period After-Tax Cash


Year Inflow/(Outflow)
0 $(600,000)
1 (500,000)
2 450,000
3 450,000
4 350,000
5 250,000

Management anticipates the equipment will be sold at the beginning of Year 6 for $50,000 and its book value will be zero.
Jones’ internal hurdle and effective income tax rates are 14% and 40%, respectively. Based on this information, a negative net
present value was computed for the project. Accordingly, it can be concluded tha.

A. The project has an internal rate of return (IRR) less than 14% since IRR is the interest rate at which net present value is
equal to zero.
B. Jones should examine the determinants of its hurdle rate further before analyzing any other potential projects.
C. Jones should calculate the project payback to determine if it is consistent with the net present value calculation.
D. The project has an IRR greater than 14% since IRR is the interest rate at which net present value is equal to zero.

[Fact Pattern #19]


Hobart Corporation evaluates capital projects using a variety of performance screens, including a hurdle rate of 16%, payback period
of 3 years or less, and an accounting rate of return of 20% or more.

Copyright 2008 Gleim Publications, Inc. Page 55


Printed for Bahaa Hassan
Gleim CMA Test Prep: Part 2: Financial Decision Making
(283 questions)

[193] (Refers to Fact Pattern #19)


Hobart’s management is completing review of a project on the basis of the following projections:

Capital investment $200,000


Annual cash flows $74,000
Straight-line depreciation 5 years
Terminal value $20,000

The projected internal rate of return is 20%. Which one of the following alternatives reflects the appropriate conclusions for
the indicated evaluative measures.

Internal Rate
of Return Payback
A. Accept Reject
B. Reject Reject
C. Accept Accept
D. Reject Accept

[194] Despite its shortcomings, the traditional payback period continues to be a popular method to evaluate investments because, in
part, i.

A. Provides some insight into the risk associated with a project.


B. Ignores the time value of money.
C. Focuses on income rather than cash flow.
D. Furnishes information about an investment’s lifetime performance.

[195] Which one of the following is not a shortcoming of the payback method.

A. It offers no consideration of cash flows beyond the expiration of the payback period.
B. It ignores the time value of money.
C. It offers no indication of a project’s liquidity.
D. It encourages establishing a short payback period.

Copyright 2008 Gleim Publications, Inc. Page 56


Printed for Bahaa Hassan
Gleim CMA Test Prep: Part 2: Financial Decision Making
(283 questions)

[196] Quint Company uses the payback method as part of its analysis of capital investments. One of its projects requires a $140,000
investment and has the following projected before-tax cash flows.

Year 1 $60,000
Year 2 60,000
Year 3 60,000
Year 4 80,000
Year 5 80,000

Quint has an effective 40% tax rate. Based on these data, the after-tax payback period i.

A. 1.5 years.
B. 2.3 years.
C. 3.4 years.
D. 3.7 years.

[197] Miller, Inc., uses straight-line depreciation for both tax and financial reporting purposes. The following data relate to Machine
No. 108, which cost $400,000 and is being written off over a 5-year life.

Operating Savings in Cash


Year Income Operating Costs
1 $150,000 $230,000
2 200,000 280,000
3 225,000 305,000
4 225,000 305,000
5 175,000 255,000

All of these amounts are on a before-tax basis. Miller is subject to a 40% income tax rate. The company strives for a 12% rate
of return. The traditional payback period for Machine No. 108 would b.

A. 2.14 years.
B. 2.44 years.
C. 2.58 years.
D. 3.41 years.

[198] Barker, Inc., has no capital rationing constraint and is analyzing many independent investment alternatives. Barker should
accept all investment proposals

A. If debt financing is available for them.


B. That have positive cash flows.
C. That provide returns greater than the before-tax cost of debt.
D. That have a positive net present value.

Copyright 2008 Gleim Publications, Inc. Page 57


Printed for Bahaa Hassan
Gleim CMA Test Prep: Part 2: Financial Decision Making
(283 questions)

[199] The profitability index approach to investment analysis

A. Fails to consider the timing of project cash flows.


B. Considers only the project’s contribution to net income and does not consider cash flow effects.
C. Always yields the same accept/reject decisions for independent projects as the net present value method.
D. Always yields the same accept/reject decisions for mutually exclusive projects as the net present value method.

[200] If an investment project has a profitability index of 1.15, the

A. Project’s internal rate of return is 15%.


B. Project’s cost of capital is greater than its internal rate of return.
C. Project’s internal rate of return exceeds its net present value.
D. Net present value of the project is positive.

[201] The technique used to evaluate all possible capital projects of different dollar amounts and then rank them according to their
desirability is the

A. Profitability index method.


B. Net present value method.
C. Payback method.
D. Discounted cash flow method.

[202] The profitability index (excess present value index)

A. Represents the ratio of the discounted net cash outflows to cash inflows.
B. Is the relationship between the net discounted cash inflows less the discounted cash outflows divided by the discounted
cash outflows.
C. Is calculated by dividing the discounted profits by the cash outflows.
D. Is the ratio of the discounted net cash inflows to discounted cash outflows.

[203] The method that divides a project’s annual after-tax net income by the average investment cost to measure the estimated
performance of a capital investment is the

A. Internal rate of return method.


B. Accounting rate of return method.
C. Payback method.
D. Net present value (NPV) method.

Copyright 2008 Gleim Publications, Inc. Page 58


Printed for Bahaa Hassan
Gleim CMA Test Prep: Part 2: Financial Decision Making
(283 questions)

[204] If income tax considerations are ignored, how is depreciation handled by the following capital budgeting techniques?

Internal Accounting
Rate of Return Rate of Return Payback
A. Excluded Included Excluded
B. Included Excluded Included
C. Excluded Excluded Included
D. Included Included Included

[205] Which mutually exclusive project would you select if both are priced at $1,000 and your discount rate is 14%: Project A, with
3 annual cash flows of $1,000, Project B, with 3 years of zero cash flow followed by 3 years of $1,500 annually?

A. Project A.
B. Project B.
C. The IRRs are equal, hence you are indifferent.
D. The NPVs are equal, hence you are indifferent.

[Fact Pattern #20]


Maloney Company uses a 12% hurdle rate for all capital expenditures and has done the following analysis for four projects for the
upcoming year:

Project 1 Project 2 Project 3 Project 4


Initial outlay $4,960,000 $5,440,000 $4,000,000 $5,960,000
Annual net cash inflows:
Year 1 1,600,000 1,900,000 1,300,000 2,000,000
Year 2 1,900,000 2,500,000 1,400,000 2,700,000
Year 3 1,800,000 1,800,000 1,600,000 1,800,000
Year 4 1,600,000 1,200,000 800,000 1,300,000
Net present value 281,280 293,240 (75,960) 85,520
Profitability index 106% 105% 98% 101%
Internal rate of return 14% 15% 11% 13%

[206] (Refers to Fact Pattern #20)


Which project(s) should Maloney undertake during the upcoming year assuming it has no budget restrictions?

A. All of the projects.


B. Projects 1, 2, and 3.
C. Projects 1, 2, and 4.
D. Projects 1 and 2.

Copyright 2008 Gleim Publications, Inc. Page 59


Printed for Bahaa Hassan
Gleim CMA Test Prep: Part 2: Financial Decision Making
(283 questions)

[207] (Refers to Fact Pattern #20)


Which projects should Maloney undertake during the upcoming year if it has only $12,000,000 of investment funds available?

A. Projects 1 and 3.
B. Projects 1, 2, and 4.
C. Projects 1 and 4.
D. Projects 1 and 2.

[208] (Refers to Fact Pattern #20)


Which project(s) should Maloney undertake during the upcoming year if it has only $6,000,000 of funds available?

A. Project 3.
B. Projects 1 and 2.
C. Project 1.
D. Project 2.

[209] The recommended technique for evaluating projects when capital is rationed and there are no mutually exclusive projects from
which to choose is to rank the projects by

A. Accounting rate of return.


B. Payback.
C. Internal rate of return.
D. Profitability index.

[210] The technique that reflects the time value of money and is calculated by dividing the present value of the future net after-tax
cash inflows that have been discounted at the desired cost of capital by the initial cash outlay for the investment is called the

A. Capital rationing method.


B. Average rate of return method.
C. Profitability index method.
D. Accounting rate of return method.

Copyright 2008 Gleim Publications, Inc. Page 60


Printed for Bahaa Hassan
Gleim CMA Test Prep: Part 2: Financial Decision Making
(283 questions)

[Fact Pattern #21]


Mercken Industries is contemplating four projects, Project P, Project Q, Project R, and Project S. The capital costs and estimated
after-tax net cash flows of each independent project are listed below. Mercken’s desired after-tax opportunity cost is 12%, and the
company has a capital budget for the year of $450,000. Idle funds cannot be reinvested at greater than 12%.

Project P Project Q Project R Project S


Initial cost $200,000 $235,000 $190,000 $210,000
Annual cash flows
Year 1 $ 93,000 $ 90,000 $ 45,000 $ 40,000
Year 2 93,000 85,000 55,000 50,000
Year 3 93,000 75,000 65,000 60,000
Year 4 0 55,000 70,000 65,000
Year 5 0 50,000 75,000 75,000
Net present value $ 23,370 $ 29,827 $ 27,333 $ (7,854)
Internal rate of return 18.7% 17.6% 17.2% 10.6%
Excess present value index 1.12 1.13 1.14 0.96

[211] (Refers to Fact Pattern #21)


During this year, Mercken will choose

A. Projects P, Q, and R.
B. Projects P, Q, R, and S.
C. Projects Q and R.
D. Projects P and Q.

[212] (Refers to Fact Pattern #21)


If Mercken is able to accept only one project, the company would choose

A. Project P.
B. Project Q because it has the highest net present value.
C. Project P because it has the highest internal rate of return.
D. Project P because it has the shortest payback period.

[213] Rohan Transport is considering two alternative buses to transport people between cities that are in the Southeastern U.S., such
as Baton Rouge and Gainesville. A gas-powered bus has a cost of $55,000, and will produce end-of-year net cash flows of
$22,000 per year for 4 years. A new electric bus will cost $90,000, and will produce cash flows of $28,000 per year for 8
years. The company must provide bus service for 8 years, after which it plans to give up its franchise and to cease operating
the route. Inflation is not expected to affect either costs or revenues during the next 8 years. If Rohan Transport’s cost of
capital is 16%, by what amount will the better project increase the company’s value?

A. $6,556
B. $(14,432)
C. $13,112
D. $31,632

Copyright 2008 Gleim Publications, Inc. Page 61


Printed for Bahaa Hassan
Gleim CMA Test Prep: Part 2: Financial Decision Making
(283 questions)

[214] Mesa Company is considering an investment to open a new banana processing division. The project in question would entail
an initial investment of $45,000, and as a result of the project cash inflows of $20,000 can be expected in each of the next 3
years. The hurdle rate is 10%. What is the profitability index for the project?

A. 1.0784
B. 1.1053
C. 1.1379
D. 1.1771

[215] Flex Corporation is studying a capital acquisition proposal in which newly acquired assets will be depreciated using the
straight-line method. Which one of the following statements about the proposal would be incorrect if a switch is made to the
Modified Accelerated Cost Recovery System (MACRS)?

A. The net present value will increase.


B. The internal rate of return will increase.
C. The payback period will be shortened.
D. The profitability index will decrease.

[Fact Pattern #22]


Capital Invest, Inc., uses a 12% hurdle rate for all capital expenditures and has done the following analysis for four projects for the
upcoming year:

Project 1 Project 2 Project 3 Project 4


Initial capital outlay $200,000 $298,000 $248,000 $272,000
Annual net cash inflows
Year 1 65,000 100,000 80,000 95,000
Year 2 70,000 135,000 95,000 125,000
Year 3 80,000 90,000 90,000 90,000
Year 4 40,000 65,000 80,000 60,000
Net present value (3,798) 4,276 14,064 14,662
Profitability index 98% 101% 106% 105%
Internal rate of return 11% 13% 14% 15%

[216] (Refers to Fact Pattern #22)


Which project(s) should Capital Invest undertake during the upcoming year, assuming it has no budget restrictions?

A. All of the projects.


B. Projects 1, 2, and 3.
C. Projects 2, 3, and 4.
D. Projects 1, 3, and 4.

Copyright 2008 Gleim Publications, Inc. Page 62


Printed for Bahaa Hassan
Gleim CMA Test Prep: Part 2: Financial Decision Making
(283 questions)

[217] (Refers to Fact Pattern #22)


Which project(s) should Capital Invest undertake during the upcoming year if it has only $600,000 of funds available?

A. Projects 1 and 3.
B. Projects 2, 3, and 4.
C. Projects 2 and 3.
D. Projects 3 and 4.

[218] (Refers to Fact Pattern #22)


Which project(s) should Capital Invest undertake during the upcoming year if it has only $300,000 of capital funds available?

A. Project 1.
B. Projects 2, 3, and 4.
C. Projects 3 and 4.
D. Project 3.

[219] The technique that measures the estimated performance of a capital investment by dividing the project’s annual after-tax net
income by the average investment cost is called the

A. Bail-out payback method.


B. Internal rate of return method.
C. Profitability index method.
D. Accounting rate of return method.

[220] Woods, Inc., is considering four independent investment proposals. Woods has $3 million available for investment during the
present period. The investment outlay for each project and its projected net present value (NPV) is presented below.

Project Investment Cost NPV


I $ 500,000 $ 40,000
II 900,000 120,000
III 1,200,000 180,000
IV 1,600,000 150,000

Which of the following project options should be recommended to Woods’ management.

A. Projects I, II, and III only.


B. Projects I, II, and IV only.
C. Projects II, III, and IV only.
D. Projects III and IV only.

Copyright 2008 Gleim Publications, Inc. Page 63


Printed for Bahaa Hassan
Gleim CMA Test Prep: Part 2: Financial Decision Making
(283 questions)

[221] The technique that incorporates the time value of money by determining the compound interest rate of an investment such that
the present value of the after-tax cash inflows over the life of the investment is equal to the initial investment is called the

A. Internal rate of return method.


B. Capital asset pricing model.
C. Profitability index method.
D. Accounting rate of return method.

[222] The technique that measures the number of years required for the after-tax cash flows to recover the initial investment in a
project is called the

A. Net present value method.


B. Payback method.
C. Profitability index method.
D. Accounting rate of return method.

[223] Capital budgeting methods are often divided into two classifications: project screening and project ranking. Which one of the
following is considered a ranking method rather than a screening method?

A. Net present value.


B. Time-adjusted rate of return.
C. Profitability index.
D. Accounting rate of return.

[224] Molar, Inc., is evaluating three independent projects for the expansion of different product lines. The Finance Department has
performed an extensive analysis of each project, and the chief financial officer has indicated that there is no capital rationing
in effect. Which of the following statements are correct?

I. Reject any project with a payback period that is shorter than the company standard.
II. The project with the highest internal rate of return (IRR) exceeding the hurdle rate should be selected and the others
rejected.
III. All projects with positive net present values should be selected.
IV. Molar should reject any projects with negative IRRs.

A. I, II, and IV only.


B. I, II, III, and IV.
C. II and III only.
D. III and IV only.

Copyright 2008 Gleim Publications, Inc. Page 64


Printed for Bahaa Hassan
Gleim CMA Test Prep: Part 2: Financial Decision Making
(283 questions)

[225] Ironside Products is considering two independent projects, each requiring a cash outlay of $500,000 and having an expected
life of 10 years. The forecasted annual net cash inflows for each project and the probability distributions for these cash inflows
are as follows:

Project R Project S
Probabilities Cash Inflows Probabilities Cash Inflows
0.10 $ 75,000 0.25 $ 70,000
0.80 95,000 0.50 110,000
0.10 115,000 0.25 150,000

Ironside has decided that the project with the greatest relative risk should meet a hurdle rate of 16% and the project with less
risk should meet a hurdle rate of 12%. Given these parameters, which of the following actions should be recommended for
Ironside to undertake.

A. Reject both projects.


B. Accept Project R and reject Project S.
C. Reject Project R and accept Project S.
D. Accept both projects.

[Fact Pattern #23]


Hobart Corporation evaluates capital projects using a variety of performance screens, including a hurdle rate of 16%, payback period
of 3 years or less, and an accounting rate of return of 20% or more.

[226] (Refers to Fact Pattern #23)


Hobart’s management is completing review of a project on the basis of the following projections:

Capital investment $200,000


Annual cash flows $65,000
Straight-line depreciation 8 years
Terminal value $20,000

The projected net present value is negative $2,000. Which one of the following alternatives reflects the appropriate
conclusions for the indicated evaluative measures.

Net Present
Value Payback
A. Accept Reject
B. Reject Accept
C. Accept Accept
D. Reject Reject

Copyright 2008 Gleim Publications, Inc. Page 65


Printed for Bahaa Hassan
Gleim CMA Test Prep: Part 2: Financial Decision Making
(283 questions)

[227] Staten Corporation is considering two mutually exclusive projects. Both require an initial outlay of $150,000 and will operate
for 5 years. The cash flows associated with these projects are as follows.

Year Project X Project Y


1 $ 47,000 $ 0
2 47,000 0
3 47,000 0
4 47,000 0
5 47,000 280,000
Total $235,000 $280,000

Staten’s required rate of return is 10%. Using the net present value method, which one of the following actions would you
recommend to Staten?

A. Accept Project Y and reject Project X.


B. Accept Projects X and Y.
C. Accept Project X and reject Project Y.
D. Reject Projects X and Y.

[228] Foggy Products is evaluating two mutually exclusive projects, one requiring a $4 million initial outlay and the other a $6
million outlay. The Finance Department has performed an extensive analysis of each project. The chief financial officer has
indicated that there is no capital rationing in effect. Which of the following statements are correct?

I. Both projects should be rejected if their payback periods are longer than the company standard.
II. The project with the highest internal rate of return (IRR) should be selected (assuming both IRRs exceed the hurdle
rate).
III. The project with the highest positive net present value should be selected.
IV. Select the project with the smaller initial investment, regardless of which evaluation method is used.

A. I, II, and IV only.


B. I, II, and III only.
C. I and III only.
D. II and III only.

[229] Winston Corporation is subject to a 30% effective income tax rate and uses the net present value method to evaluate capital
budgeting proposals. Harry Ralston, the capital budget manager, desires to improve the appeal of a marginally attractive
proposal. To accomplish his goal, which one of the following actions should be recommended to Ralston?

A. Postpone a fully deductible major overhaul from Year 4 to Year 5.


B. Decrease the project’s estimated terminal salvage value.
C. Immediately pay the proposal’s marketing program in its entirety rather than pay in five equal installments.
D. Adjust the project’s discount rate to reflect movement of the project from a low risk category to an average risk
category.

Copyright 2008 Gleim Publications, Inc. Page 66


Printed for Bahaa Hassan
Gleim CMA Test Prep: Part 2: Financial Decision Making
(283 questions)

[230] Diane Harper, Vice President of Finance for BGN Industries, is reviewing material prepared by her staff prior to the board of
directors meeting at which she must recommend one of four mutually exclusive options for a new product line. The summary
information below indicates the initial investment required, the present value of cash inflows (excluding the initial investment)
at BGN’s hurdle rate of 16%, and the internal rate of return (IRR) for each of the four options.

Present Value
of Cash Inflows
Option Investment at 16% IRR
X $3,950,000 $3,800,000 15.5%
Y 3,000,000 3,750,000 19.0%
Z 2,000,000 2,825,000 17.25%
W 800,000 1,100,000 18.0%

If there are no capital rationing constraints, which option should Harper recommend.

A. Option X.
B. Option Y.
C. Option Z.
D. Option W.

[231] Wearwell Company is considering three investment projects. Wearwell’s president asked the controller to prepare a report and
recommend an appropriate investment decision. The results of the controller’s calculations for the three projects are as
follows:

Net Internal Rat.


Project Present Value of Return
A $20,680 12%
B 30,300 10%
C 15,000 13%

The company expects a minimum net present value (NPV) of $20,000 from accepted projects. The projects are mutually
exclusive, and Wearwell’s cost of capital is 8%. Which one of the following options should the controller recommend to the
president.

A. Project C because it has the highest internal rate of return (IRR).


B. Project B because it has the highest net present value (NPV).
C. Projects A, B, and C because each of the projects has an IRR greater than the cost of capital.
D. Projects A and B because they exceed the minimum expected NPV.

[232] Which one of the following capital budgeting techniques would result in the same project selection as the net present value
method.

A. Discounted payback.
B. Internal rate of return.
C. Profitability index.
D. Accounting rate of return.

Copyright 2008 Gleim Publications, Inc. Page 67


Printed for Bahaa Hassan
Gleim CMA Test Prep: Part 2: Financial Decision Making
(283 questions)

[233] Lewis Services is evaluating six investment opportunities (projects). The following table reflects each project’s net present
value (NPV) and the respective initial investments required. All of these projects are independent.

Project NPV Investment


R $ 5,000 $10,000
S 5,000 5,000
T 8,000 40,000
U 15,000 60,000
V 15,000 75,000
W 3,000 15,000

Lewis has an investment constraint of $100,000. Which combination of projects would represent the optimal investment that
should be recommended to Lewis Services’ management.

A. R, S, U, and W.
B. R, V, and W.
C. R, S, and V.
D. T and U.

[234] Zinx Corporation has a maximum of $5,000,000 available for investments. The company has identified the following
investment options:

Discounted
Project Investment Cash Flow
I $2,800,000 $3,360,000
II 1,500,000 1,720,000
III 2,300,000 2,617,000
IV 1,200,000 1,368,000
V 800,000 1,000,000

Which of the following project alternatives should be recommended to Zinx’s management.

A. II, III, and IV.


B. II, III, and V.
C. I and II.
D. I, IV, and V.

[235] In evaluating independent capital investment projects, the best reason for a firm to accept such projects is a(n.

A. Accounting rate of return greater than zero.


B. Initial investment greater than the present value of cash inflows.
C. Profitability index greater than one.
D. Internal rate of return greater than the accounting rate of return.

Copyright 2008 Gleim Publications, Inc. Page 68


Printed for Bahaa Hassan
Gleim CMA Test Prep: Part 2: Financial Decision Making
(283 questions)

[236] Carbide, Inc., has the following investment opportunities. Required investment outlays and the profitability index for each of
these investments are as follows:

Project Investment Cost Profitability Index


I $300,000 0.5
II 450,000 1.4
III 650,000 1.8
IV 750,000 1.6

Carbide’s budget ceiling for initial outlays during the present period is $1,500,000. The proposed projects are independent of
each other. Which project or projects would you recommend that Carbide accept.

A. III.
B. III and IV.
C. I, II, and IV.
D. I, III, and IV.

[237] Dobson Corp. is analyzing a capital investment requiring a cash outflow at time = 0 of $2.5 million and net cash inflows of
$800,000 per year for 5 years. The net present value (NPV) was calculated to be $384,000 at a 12% discount rate. Since
several managers felt this was a risky project, three separate scenarios were analyzed, as follows:

Scenario R - The annual cash inflows were reduced by 10%.


Scenario S - The discount rate was changed to 18%.
Scenario T - The cash inflow in Year 5 was reduced to zero.

Rank the three individual scenarios in the order of the effect on NPV, from least effect to greatest effect.

A. R, S, T.
B. R, T, S.
C. S, T, R.
D. T, S, R.

[238] Monroe Company needs an additional machine that will be used for the next 5 years, at which time the machine will be
obsolete and have zero salvage value. Monroe has two options available: purchase the asset for the list price of $300,000 cash
or lease the asset, requiring five annual lease payments of $68,000 with the first payment due immediately. The lease
payments include 6% interest. Excluding depreciation considerations, the best alternative is to

A. Purchase the asset for a $40,000 advantage.


B. Purchase the asset for a $3,627 advantage.
C. Lease the asset for a $45,918 advantage.
D. Lease the asset for a $13,557 advantage.

Copyright 2008 Gleim Publications, Inc. Page 69


Printed for Bahaa Hassan
Gleim CMA Test Prep: Part 2: Financial Decision Making
(283 questions)

[239] If a project has a profitability index that is greater than 1.0, it means that the

A. Initial investment exceeds the cash flows.


B. Cash flows exceed the initial investment.
C. Required return is less than the internal rate of return.
D. Internal rate of return is equal to the required return.

[240] Large firms often seek to control risk through allocating or rationing capital among divisions. When capital is rationed,
managers are most likely to choose among prospective investments based on their

A. Net present value (NPV) rankings.


B. Internal rate of return (IRR) rankings.
C. Payback periods.
D. Profitability index rankings.

[241] Using the profitability index, which one of the following projects is the best investment?

Project A Project B Project C Project D


Initial investment $10,000,000 $7,000,000 $5,000,000 $2,000,000
Net present value 1,100,000 700,000 300,000 250,000
Internal rate of
return 11% 12% 8% 10%

A. Project A.
B. Project B.
C. Project C.
D. Project D.

[Fact Pattern #24]


McLean, Inc., is considering the purchase of a new machine that will cost $150,000. The machine has an estimated useful life of 3
years. Assume that 30% of the depreciable base will be depreciated in the first year, 40% in the second year, and 30% in the third
year. The new machine will have a $10,000 resale value at the end of its estimated useful life. The machine is expected to save the
company $85,000 per year in operating expenses. McLean uses a 40% estimated income tax rate and a 16% hurdle rate to evaluate
capital projects.
Discount rates for a 16% rate are as follows:

Present Value of an
Present Value of $1 Ordinary Annuity of $1
Year 1 .862 .862
Year 2 .743 1.605
Year 3 .641 2.246

Copyright 2008 Gleim Publications, Inc. Page 70


Printed for Bahaa Hassan
Gleim CMA Test Prep: Part 2: Financial Decision Making
(283 questions)

[242] (Refers to Fact Pattern #24)


What is the net present value of this project?

A. $15,842
B. $13,278
C. $40,910
D. $9,432

[243] (Refers to Fact Pattern #24)


The payback period for this investment would be

A. 2.94 years.
B. 1.76 years.
C. 2.09 years.
D. 1.14 years.

[Fact Pattern #25]


MS Trucking is considering the purchase of a new piece of equipment that has a net initial investment with a present value of
$300,000. The equipment has an estimated useful life of 3 years. For tax purposes, the equipment will be fully depreciated at rates of
30%, 40%, and 30% in Years 1, 2, and 3, respectively. The new machine is expected to have a $20,000 salvage value. The machine is
expected to save the company $170,000 per year in operating expenses. MS Trucking has a 40% marginal income tax rate and a 16%
cost of capital. Discount rates for a 16% rate are as follows:

Present Value of an
Ordinary Annuity of $1 Present Value of $1
Year 1 .862 .862
Year 2 1.605 .743
Year 3 2.246 .641

[244] (Refers to Fact Pattern #25)


What is the net present value of this project?

A. $81,820
B. $26,556
C. $118,956
D. $138,000

Copyright 2008 Gleim Publications, Inc. Page 71


Printed for Bahaa Hassan
Gleim CMA Test Prep: Part 2: Financial Decision Making
(283 questions)

[245] (Refers to Fact Pattern #25)


What is the profitability index for the project?

A. 1.089
B. 0.789
C. 1.315
D. 1.059

[246] (Refers to Fact Pattern #25)


The payback period for this investment is

A. 2.08 years.
B. 2.79 years.
C. 2.09 years.
D. 3.00 years.

[Fact Pattern #26]


A proposed investment is not expected to have any salvage value at the end of its 5-year life. Because of realistic depreciation
practices, the net carrying amount and the salvage value are equal at the end of each year. For present value purposes, cash flows are
assumed to occur at the end of each year. The company uses a 12% after-tax target rate of return.

Purchase Cost and Annual Net After-Tax


Year Carrying Amount Cash Flows Annual Net Income
0 $500,000 $ 0 $ 0
1 336,000 240,000 70,000
2 200,000 216,000 78,000
3 100,000 192,000 86,000
4 36,000 168,000 94,000
5 0 144,000 102,000

Discount Factors for a 12% Rate of Return


Present Value of $1 at Present Value of an Annuity of
Year the End of Each Period $1 at the End of Each Period
1 .89 .89
2 .80 1.69
3 .71 2.40
4 .64 3.04
5 .57 3.61
6 .51 4.12

Copyright 2008 Gleim Publications, Inc. Page 72


Printed for Bahaa Hassan
Gleim CMA Test Prep: Part 2: Financial Decision Making
(283 questions)

[247] (Refers to Fact Pattern #26)


The accounting rate of return based on the average investment is

A. 84.9%
B. 34.4%
C. 40.8%
D. 12%

[248] (Refers to Fact Pattern #26)


The net present value is

A. $304,060
B. $212,320
C. $(70,000)
D. $712,320

[249] (Refers to Fact Pattern #26)


The profitability index is

A. .608
B. .425
C. .860
D. 1.425

[250] (Refers to Fact Pattern #26)


Which statement about the internal rate of return of the investment is true?

A. The IRR is exactly 12%.


B. The IRR is over 12%.
C. The IRR is under 12%.
D. No information about the IRR can be determined.

Copyright 2008 Gleim Publications, Inc. Page 73


Printed for Bahaa Hassan
Gleim CMA Test Prep: Part 2: Financial Decision Making
(283 questions)

[Fact Pattern #27]


Yipann Corporation is reviewing an investment proposal. The initial cost, as well as other related data for each year, are presented in
the schedule below. All cash flows are assumed to take place at the end of the year. The salvage value of the investment at the end of
each year is equal to its net book value, and there will be no salvage value at the end of the investment’s life.

Investment Proposal
Annual
Initial Cost Net After-Tax Annual
Year and Book Value Cash Flows Net Income
0 $105,000 $ 0 $ 0
1 70,000 50,000 15,000
2 42,000 45,000 17,000
3 21,000 40,000 19,000
4 7,000 35,000 21,000
5 0 30,000 23,000

Yipann uses a 24% after-tax target rate of return for new investment proposals. The discount factors for a 24% rate of return are
given.

Present Value of
Present Value of an Annuity of $1.00
$1.00 Received at Received at the End
Year the End of Period of Each Period
1 .81 .81
2 .65 1.46
3 .52 1.98
4 .42 2.40
5 .34 2.74
6 .28 3.02
7 .22 3.24

[251] (Refers to Fact Pattern #27)


The average annual cash inflow at which Yipann would be indifferent to the investment (rounded to the nearest dollar) is

A. $21,000
B. $40,000
C. $38,321
D. $46,667

[252] (Refers to Fact Pattern #27)


The accounting rate of return for the investment proposal over its life using the initial value of the investment is

A. 36.2%
B. 18.1%
C. 28.1%
D. 38.1%

Copyright 2008 Gleim Publications, Inc. Page 74


Printed for Bahaa Hassan
Gleim CMA Test Prep: Part 2: Financial Decision Making
(283 questions)

[253] (Refers to Fact Pattern #27)


The net present value of the investment proposal is

A. $4,600
B. $10,450
C. $(55,280)
D. $115,450

[254] (Refers to Fact Pattern #27)


The traditional payback period for the investment proposal is

A. .875 years.
B. 1.833 years.
C. 2.250 years.
D. More than 5 years.

[Fact Pattern #28]


Rex Company is considering an investment in a new plant, which will entail an immediate capital expenditure of $4,000,000. The
plant is to be depreciated on a straight-line basis over 10 years to zero salvage value. Operating income (before depreciation and
taxes) is expected to be $800,000 per year over the 10-year life of the plant. The opportunity cost of capital is 14%. Assume that there
are no taxes.

[255] (Refers to Fact Pattern #28)


What is the book (or accounting) rate of return for the investment using the average investment method?

A. 10%
B. 20%
C. 28%
D. 35%

[256] (Refers to Fact Pattern #28)


What is the discounted payback period for the investment?

A. 5.5 years.
B. 7.1 years.
C. 9.2 years.
D. 11.7 years.

Copyright 2008 Gleim Publications, Inc. Page 75


Printed for Bahaa Hassan
Gleim CMA Test Prep: Part 2: Financial Decision Making
(283 questions)

[257] (Refers to Fact Pattern #28)


What is the NPV for the investment?

A. $172,800
B. $(1,913,600)
C. $520,000
D. $362,400

[Fact Pattern #29]


Don Adams Breweries is considering an expansion project with an investment of $1,500,000. The equipment will be depreciated to
zero salvage value on a straight-line basis over 5 years. The expansion will produce incremental operating revenue of $400,000
annually for 5 years. The company’s opportunity cost of capital is 12%. Ignore taxes.

[258] (Refers to Fact Pattern #29)


What is the NPV of the investment?

A. $0
B. $(58,000)
C. $(116,000)
D. $1,442,000

[259] (Refers to Fact Pattern #29)


What is the IRR of the investment?

A. 10.43%
B. 12.68%
C. 16.32%
D. 19.17%

[260] (Refers to Fact Pattern #29)


What is the book (accounting) rate of return of the investment using the average investment method?

A. 6.67%
B. 13.33%
C. 16.67%
D. 26.67%

Copyright 2008 Gleim Publications, Inc. Page 76


Printed for Bahaa Hassan
Gleim CMA Test Prep: Part 2: Financial Decision Making
(283 questions)

[261] (Refers to Fact Pattern #29)


What is the payback period of the project?

A. 2 years.
B. 2.14 years.
C. 3.75 years.
D. 5 years.

[Fact Pattern #30]


Tonya, Inc., has a cost of capital of 15% and is considering the acquisition of a new machine that costs $800,000 and has a useful life
of 5 years. Tonya projects that earnings and cash flow will increase as follows:

Year Net Earnings After-Tax Cash Flow


1 $200,000 $320,000
2 200,000 280,000
3 200,000 200,000
4 200,000 200,000
5 200,000 200,000

Interest rate factors at 15% are as follows:

Present Value
Period Present Value of $1 of an Annuity
1 .87 0.87
2 .76 1.63
3 .66 2.29
4 .57 2.86
5 .50 3.36

[262] (Refers to Fact Pattern #30)


What is the payback period of this investment?

A. 1.5 years.
B. 3.0 years.
C. 3.3 years.
D. 4.0 years.

[263] (Refers to Fact Pattern #30)


The net present value of this investment is

A. $(128,000)
B. $200,000
C. $37,200
D. $400,000

Copyright 2008 Gleim Publications, Inc. Page 77


Printed for Bahaa Hassan
Gleim CMA Test Prep: Part 2: Financial Decision Making
(283 questions)

[264] (Refers to Fact Pattern #30)


What is the profitability index for the investment?

A. 0.50
B. 0.96
C. 1.05
D. 1.25

[Fact Pattern #31]


The Keego Company is planning a $200,000 equipment investment that has an estimated 5-year life with no estimated salvage value.
The company has projected the following annual cash flows for the investment:

Projected
Year Cash Inflows Present Value of $1
1 $120,000 .91
2 60,000 .76
3 40,000 .63
4 40,000 .53
5 40,000 .44
Totals $300,000 3.27

[265] (Refers to Fact Pattern #31)


Assuming that the estimated cash inflows occur evenly during each year, the payback period for the investment is

A. 1.67 years.
B. 4.91 years.
C. 2.50 years.
D. 1.96 years.

[266] (Refers to Fact Pattern #31)


The net present value for the investment is

A. $18,800
B. $218,800
C. $100,000
D. $91,743

Copyright 2008 Gleim Publications, Inc. Page 78


Printed for Bahaa Hassan
Gleim CMA Test Prep: Part 2: Financial Decision Making
(283 questions)

[Fact Pattern #32]


Willis, Inc., has a desired rate of return of 15% and is considering the acquisition of a new machine that costs $400,000 and has a
useful life of 5 years. Willis projects that earnings and cash flow will increase as follows:

Net After-Tax 15% Interest Rate Factors


Year Earnings Cash Flow Present Present Value of
1 $100,000 $160,000 Period Value of $1 an Annuity of $1
2 100,000 140,000 1 0.87 0.87
3 100,000 100,000 2 0.76 1.63
4 100,000 100,000 3 0.66 2.29
5 200,000 100,000 4 0.57 2.86
5 0.50 3.36

[267] (Refers to Fact Pattern #32)


What is the payback period of Willis’ investment?

A. 1.5 years.
B. 3.0 years.
C. 3.3 years.
D. 4.0 years.

[268] (Refers to Fact Pattern #32)


The net present value of Willis’ investment is

A. Negative, $64,000.
B. Negative, $14,000.
C. Positive, $18,600.
D. Positive, $200,000.

[269] For capital budgeting purposes, management would select a high hurdle rate of return for certain projects because
management

A. Wants to use equity funding exclusively.


B. Believes too many proposals are being rejected.
C. Believes bank loans are riskier than capital investments.
D. Wants to factor risk into its consideration of projects.

Copyright 2008 Gleim Publications, Inc. Page 79


Printed for Bahaa Hassan
Gleim CMA Test Prep: Part 2: Financial Decision Making
(283 questions)

[270] A company uses portfolio theory to develop its investment portfolio. If the company wishes to obtain optimal risk reduction
through the portfolio effect, it should make its next investment in an investment that

A. Correlates negatively to the current portfolio holdings.


B. Is uncorrelated to the current portfolio holdings.
C. Is highly correlated to the current portfolio holdings.
D. Is perfectly correlated to the current portfolio holdings.

[271] Mega, Inc., a large conglomerate with operating divisions in many industries, uses risk-adjusted discount rates in evaluating
capital investment decisions. Consider the following statements concerning Mega’s use of risk-adjusted discount rates.

I. Mega may accept some investments with internal rates of return less than Mega’s overall average cost of capital.
II. Discount rates vary depending on the type of investment.
III. Mega may reject some investments with internal rates of return greater than the cost of capital.
IV. Discount rates may vary depending on the division.

Which of the above statements are correct?

A. I and III only.


B. II and IV only.
C. II, III, and IV only.
D. I, II, III, and IV.

[272] Sensitivity analysis, if used with capital projects,

A. Is used extensively when cash flows are known with certainty.


B. Measures the change in the discounted cash flows when using the discounted payback method rather than the net
present value method.
C. Is a “what-if” technique that asks how a given outcome will change if the original estimates of the capital budgeting
model are changed.
D. Is a technique used to rank capital expenditure requests.

[273] An analysis of a company’s planned equity financing using the Capital Asset Pricing Model (or Security Market Line)
incorporates only the

A. Expected market earnings, the current U.S. Treasury bond yield, and the beta coefficient.
B. Expected market earnings and the price-earnings ratio.
C. Current U.S. Treasury bond yield, the price-earnings ratio, and the beta coefficient.
D. Current U.S. Treasury bond yield and the dividend payout ratio.

[274] The proper discount rate to use in calculating certainty equivalent net present value is the

A. Risk-adjusted discount rate.


B. Cost of capital.
C. Risk-free rate.
D. Cost of equity capital.

Copyright 2008 Gleim Publications, Inc. Page 80


Printed for Bahaa Hassan
Gleim CMA Test Prep: Part 2: Financial Decision Making
(283 questions)

[275] When the risks of the individual components of a project’s cash flows are different, an acceptable procedure to evaluate these
cash flows is to

A. Divide each cash flow by the payback period.


B. Compute the net present value of each cash flow using the firm’s cost of capital.
C. Compare the internal rate of return from each cash flow to its risk.
D. Discount each cash flow using a discount rate that reflects the degree of risk.

[276] A manager wants to know the effect of a possible change in cash flows on the net present value of a project. The technique
used for this purpose is

A. Sensitivity analysis.
B. Risk analysis.
C. Cost behavior analysis.
D. Return on investment analysis.

[277] A widely used approach that is used to recognize uncertainty about individual economic variables while obtaining an
immediate financial estimate of the consequences of possible prediction errors is

A. Expected value analysis.


B. Learning curve analysis.
C. Sensitivity analysis.
D. Regression analysis.

[278] Sensitivity analysis is used in capital budgeting to

A. Estimate a project’s internal rate of return.


B. Determine the amount that a variable can change without generating unacceptable results.
C. Simulate probabilistic customer reactions to a new product.
D. Identify the required market share to make a new product viable and produce acceptable results.

[279] When determining net present value in an inflationary environment, adjustments should be made to

A. Increase the discount rate only.


B. Increase the estimated cash inflows and increase the discount rate.
C. Increase the estimated cash inflows but not the discount rate.
D. Decrease the estimated cash inflows and increase the discount rate.

Copyright 2008 Gleim Publications, Inc. Page 81


Printed for Bahaa Hassan
Gleim CMA Test Prep: Part 2: Financial Decision Making
(283 questions)

[280] When evaluating a capital budgeting project, a company’s treasurer wants to know how changes in operating income and the
number of years in the project’s useful life will affect its breakeven internal rate of return. The treasurer is most likely to us.

A. Scenario analysis.
B. Sensitivity analysis.
C. Monte Carlo simulation.
D. Learning curve analysis.

[Fact Pattern #33]

A company is evaluating the possible introduction of a new


version of an existing product that will have a 2-year life cycle. (Refer to Figure FIGURE03_08_01_Q.)
At the end of 2 years, this version will be obsolete, with no
additional cash flows or salvage value. The initial and sole The following interest factors for the present value of $1 at
outlay for the modified product is $6 million, and the 10% are relevant:
company’s desired rate of return is 10%. Following are the
potential cash flows (assumed to occur at the end of each year)
Period 1 .909
and their probabilities if the product is marketed:
2 .826

[281] (Refers to Fact Pattern #33)


The project’s net present value is

A. $878,050
B. $3,242,050
C. $3,636,000
D. $6,000,000

[282] (Refers to Fact Pattern #33)


Assume the company has the real option to abandon the project at the end of Year 1. If the salvage value at that time is $3
million and the desired rate of return remains at 10%, what is the project’s net present value?

A. $878,050
B. $1,200,550
C. $2,746,450
D. $4,454,100

[283] Which of the following is not an example of a real option in a capital budgeting decision?

A. Abandonment.
B. Follow-up investment.
C. Option to wait and learn.
D. Risk-adjusted discount rates.

Copyright 2008 Gleim Publications, Inc. Page 82


Printed for Bahaa Hassan
Gleim CMA Test Prep: Part 2: Financial Decision Making
(283 questions)

Copyright 2008 Gleim Publications, Inc. Page 83


Printed for Bahaa Hassan

You might also like